You are on page 1of 63

CARDIO PACKRAT

Study online at quizlet.com/_23pihf


1.

History & Physical/Cardiology


Which of the following conditions would cause a positive
Kussmaul's sign on physical examination?
Answers
A. Left ventricular failure
B. Pulmonary edema
C. Coarctation of the aorta
D. Constrictive pericarditis

Explanations
(u) A. Left ventricular failure results in the back-up of blood into
the left atrium and then the pulmonary system so it would not
be associated with Kussmaul's sign.
(u) B. Pulmonary edema primarily results in increased
pulmonary pressures rather than having effects on the venous
inflow into the heart.
(u) C. Coarctation of the aorta primarily affects outflow from the
heart due to the stenosis resulting in delayed and decreased
femoral pulses; it has no effect on causing Kussmaul's sign.
(c) D. Kussmaul's sign is an increase rather than the normal
decrease in the CVP during inspiration. It is most often caused
by severe right-sided heart failure; it is a frequent finding in
patients with constrictive pericarditis or right ventricular
infarction.

2.

History & Physical/Cardiology


Anginal chest pain is most commonly described as which of the
following?
Answers
A. Pain changing with position or respiration
B. A sensation of discomfort
C. Tearing pain radiating to the back
D. Pain lasting for several hours

Explanations
(u) A. Pain changing with position or respiration is suggestive of
pericarditis.
(c) B. Myocardial ischemia is often experienced as a sensation of
discomfort lasting 5-15 minutes, described as dull, aching or
pressure.
(u) C. Tearing pain with radiation to the back represents aortic
dissection.
(u) D. Chest pain lasting for several hours is more suggestive for
myocardial infarction.

3.

History & Physical/Cardiology


Eliciting a history from a patient presenting with dyspnea due to
early heart failure the severity of the dyspnea should be
quantified by
Answers
A. amount of activity that precipitates it.
B. how many pillows they sleep on at night.
C. how long it takes the dyspnea to resolve.
D. any associated comorbidities.

Explanations
(c) A. The amount of activity that precipitates dyspnea should
be quantified in the history.
(u) B. Orthopnea or paroxysmal nocturnal dyspnea can be
quantified by how many pillows a patient needs to sleep on to
be comfortable.
(u) C. How long dyspnea takes to resolve or associated
comorbidities has no bearing on quantifying the severity of
dyspnea.
(u) D. See answer C above.

4.

History & Physical/Cardiology


A 25 year-old female presents with a three-day history of chest
pain aggravated by coughing and relieved by sitting. She is febrile
and a CBC with differential reveals leukocytosis. Which of the
following physical exam signs is characteristic of her problem?
Answers
A. Pulsus paradoxus
B. Localized crackles
C. Pericardial friction rub
D. Wheezing

Explanations
(u) A. Pulsus paradoxus is a classic finding for cardiac
tamponade.
(u) B. Localized crackles are associated with pneumonia and
consolidation, not pericarditis.
(c) C. Pericardial friction rub is characteristic of an inflammatory
pericarditis.
(u) D. Wheezing is characteristic for pulmonary disorders, such
as asthma.

5.

History & Physical/Cardiology


A 65 year-old white female presents with dilated tortuous veins on
the medial aspect of her lower extremities. Which of the following
would be the most common initial complaint?
Answers
A. Pain in the calf with ambulation
B. Dull aching heaviness brought on by periods of standing
C. Brownish pigmentation above the ankle
D. Edema in the lower extremities

Explanations
(u) A. Patients with deep venous thrombosis (DVT) may
present with complaints of pain in the calf with ambulation.
Secondary varicosities may result from DVT's.
(c) B. Dull aching heaviness or a feeling of fatigue brought on
by periods of standing is the most common complaint of
patients presenting initially with varicosities.
(u) C. Stasis Dermatitis and edema are most suggestive of
chronic venous insufficiency.
(u) D. See C for explanation.

6.

History & Physical/Cardiology


A 22 year-old male received a stab wound in the chest an hour ago.
The diagnosis of pericardial tamponade is strongly supported by
the presence of
Answers
A. pulmonary edema.
B. wide pulse pressure.
C. distended neck veins.
D. an early diastolic murmur.

Explanations
(u) A. Pulmonary edema may result with low output states as
seen with myocardial contusions, but it is not strongly
suggestive of tamponade.
(u) B. Wide pulse pressure is seen in conditions of high stroke
volume such as aortic insufficiency or hyperthyroidism.
Narrow pulse pressure is seen with cardiac tamponade.
(c) C. Cardiac compression will manifest with distended neck
veins and cold clammy skin.
(u) D. The onset of diastolic murmur is suggestive of valvular
disease, not tamponade.

7.

Diagnostic Studies/Cardiology
Cardiac nuclear scanning is done to detect
Answers
A. electrical conduction abnormalities.
B. valvular abnormalities.
C. ventricular wall dysfunction.
D. coronary artery patency/occlusion.

Explanations
(u) A. An EKG is used to determine electrical conduction
abnormalities.
(u) B. An echocardiogram is a non-invasive test used to
determine valvular abnormalities and wall motion.
(c) C. Visualization of the cardiac wall can be done with cardiac
nuclear scanning. This is done to determine hypokinetic areas
from akinetic areas.
(u) D. Patency or occlusion is assessed with cardiac
catheterization (invasive).

8.

Diagnostic Studies/Cardiology
A 72 year-old male with a new diagnosis of congestive heart
failure and atrial fibrillation, develops episodes of
hemodynamic compromise secondary to increased
ventricular rate. A decision to perform elective
cardioversion is made and the patient is anticoagulated
with heparin. Which test should be ordered to assess for
atrial or ventricular mural thrombi?

(u) A. Electrical conduction will not assess for mural thrombi.


(u) B. A chest x-ray will not visualize the left atria and ventricles to
assess for mural thrombi.
(c) C. Transesophageal echocardiography allows for determination of
mural thrombi that may have resulted from atrial fibrillation.
(u) D. C-reactive protein is not going to give you any information
regarding thrombi. This test is used to identify the presence of
inflammation.

A. Electrocardiogram
B. Chest x-ray
C. Transesophageal Echocardiogram
D. C-reactive protein
9.

Diagnostic Studies/Cardiology
A 64 year-old patient with known history of type 1 diabetes
mellitus for 50 years has developed pain radiating from the
right buttock to the calf. Patient states that the pain is
made worse with walking and climbing stairs. Based upon
this history which of the following would be the most
appropriate test to order?
Answers
A. Venogram
B. Arterial duplex scanning
C. X-ray of the right hip and L/S spine
D. Venous Doppler ultrasound

Explanations
(u) A. See B for explanation.
(c) B. Given the patient's long history of type 1 diabetes mellitus the
patient most likely has vascular occlusive disease. Evaluation of arterial
blood flow is assessed using the duplex scanner. X-ray of the L/S spine
and right hip while not harmful may give information regarding bony
structures. Venous Doppler ultrasound will not give information of
arterial perfusion.
(u) C. See B for explanation.
(u) D. See B for explanation.

10.

Diagnostic Studies/Cardiology
A 36 year-old male complains of occasional episodes of
"heart fluttering". The patient describes these episodes as
frequent, short-lived and episodic. He denies any
associated chest pain. Based on this information, which
one of the following tests would be the most appropriate
to order?
Answers
A. Holter monitor
B. Cardiac catheterization
C. Stress testing
D. Cardiac nuclear scanning

Explanations
(c) A. Holter monitoring is a non-invasive test done to obtain a
continuous monitoring of the electrical activity of the heart. This can
help to detect cardiac rhythm disturbances that can correlate with the
patient symptoms. Cardiac catheterization is an invasive procedure
done to assess coronary artery disease. Stress testing and cardiac
nuclear scanning are non-invasive testing maneuvers done to assess
coronary artery disease.
(u) B. See A for explanation.
(u) C. See A for explanation.
(u) D. See A for explanation.

11.

Diagnostic Studies/Cardiology
A patient with a mitral valve replacement was placed postoperatively on warfarin (Coumadin) for anticoagulation
prophylaxis. To monitor this drug for its effectiveness,
what test would be used?
Answers
A. PTT
B. PT-INR
C. Platelet aggregation
D. Bleeding time

Explanations
(u) A. PTT is a reflection of the intrinsic clotting system and is used to
monitor heparin administration.
(c) B. PT-INR is a reflection of the extrinsic and common pathway
clotting system. Coumadin interferes with Vitamin K synthesis which is
needed in the manufacture of factors II, VII, IX, X which are part of the
extrinsic clotting pathway.
(u) C. Platelet aggregation tests are utilized to assess platelet
dysfunction.
(u) D. Bleeding time is used to assess platelet function.

12.

Diagnosis/Cardiology
A 64 year-old male, with a long history of COPD, presents with increasing fatigue over the last
three months. The patient has stopped playing golf and also complains of decreased appetite,
chronic cough and a bloated feeling. Physical examination reveals distant heart sounds,
questionable gallop, lungs with decreased breath sounds at lung bases and the abdomen reveals
RUQ tenderness with the liver two finger-breadths below the costal margin, the extremities show
2+/4+ pitting edema. Labs reveal the serum creatinine level 1.6 mg/dl, BUN 42 mg/dl, liver function
test's mildly elevated and the CBC to be normal. Which of the following is the most likely
diagnosis?
Answers
A. Right ventricular failure
B. Pericarditis
C. Exacerbation of COPD
D. Cirrhosis

Explanations
(c) A. Signs of right
ventricular failure are fluid
retention i.e. edema,
hepatic congestion and
possibly ascites.
(u) B. See A for explanation.
(u) C. See A for explanation.
(u) D. See A for explanation.

13.

Diagnosis/Cardiology
A 56 year-old male with a known history of polycythemia suddenly complains of pain and
paresthesia in the left leg. Physical examination reveals the left leg is cool to the touch and the
toes are cyanotic. The popliteal pulse is absent by palpation and Doppler. The femoral pulse is
absent by palpation but weak with Doppler. The right leg and upper extremities has 2+/4+ pulses
throughout. Given these findings what is the most likely diagnosis?
Answers
A. Venous thrombosis
B. Arterial thrombosis
C. Thromboangiitis obliterans
D. Thrombophlebitis

Explanations
(u) A. See B for explanation.
(c) B. Arterial thrombosis
has occurred and is
evidenced by the loss of the
popliteal and dorsalis pedis
pulse. This is a surgical
emergency. Venous
occlusion and
thrombophlebitis do not
result in loss of arterial
pulse.
(u) C. See B for explanation.
(u) D. See B for explanation.

14.

Diagnosis/Cardiology
A 48 year-old male with a known history of hypertension is brought to the ED complaining of
headache, general malaise, nausea and vomiting. The patient currently takes nifedipine
(Procardia)90mg XL every day and atenolol (Tenormin) 50 mg every day. Vital signs reveal
temperature 98.6F, pulse 72/minute, respiratory rate 20/minute, and the blood pressure is
168/120 mmHg. BP reading taken every 15 minutes from the time of admission reveal the systolic
to run from 176 to 186 mmHg and the diastolic to run from 135 to 150 mmHg. Physical
examination reveals papilledema bilaterally. There are no renal bruits noted. The EKG is normal.
Based upon this presentation, what is the most likely diagnosis?
Answers
A. Meningitis
B. Secondary hypertension
C. Pseudotumor cerebri
D. Malignant hypertension

Explanations
(u) A. See D for explanation.
(u) B. See D for explanation.
(u) C. Pseudotumor cerebri
presents with papilledema,
but not hypertension and is
more common in young
females.
(c) D. Malignant HTN is
characterized by diastolic
reading greater than 140
mm Hg with evidence of
target organ damage.

15.

Diagnosis/Cardiology
A 55 year-old male is seen in follow-up for a complaint of chest pain. Patient states
that he has had this chest pain for about one year now. The patient further states that
the pain is retrosternal with radiation to the jaw. "It feels as though a tightness, or
heaviness is on and around my chest". This pain seems to come on with exertion
however, over the past two weeks he has noticed that he has episodes while at rest. If
the patient remains non- active the pain usually resolves in 15-20 minutes. Patient has
a 60-pack year smoking history and drinks a martini daily at lunch. Patient appears
overweight on inspection. Based upon this history what is the most likely diagnosis?
Answers
A. Acute myocardial infarction
B. Prinzmetal variant angina
C. Stable angina
D. Unstable angina

Explanations
(u) A. Pain does not resolve in an acute
MI, it gradually gets worse.
(u) B. Pain typically occurs at rest is one of
the hallmarks of Prinzmetal variant
angina. This patient has just started to
develop pain at rest.
(u) C. Pain in stable angina is relieved
with rest and usually resolves within 10
minutes. angina does not have pain at
rest.
(c) D. Pain in unstable angina is
precipitated by less effort than before or
occurs at rest.

16.

Diagnosis/Cardiology
Stable
A 60 year-old male is brought to the ED complaining of severe onset of chest pain and
intrascapular pain. The patient states that the pain feels as though "something is
ripping and tearing". The patient appears shocky; the skin is cool and clammy. The
patient has an impaired sensorium. Physical examination reveals a loud diastolic
murmur and variation in blood pressure between the right and left arm. Based upon
this presentation what is the most likely diagnosis?
Answers
A. Aortic dissection
B. Acute myocardial infarction
C. Cardiac tamponade
D. Pulmonary embolism

Explanations
(c) A. The scenario presented here is
typical of an ascending aortic dissection.
In an acute myocardial infarction the
pain builds up gradually. Cardiac
tamponade may occur with a dissection
into the pericardial space; syncope is
usually seen with this occurrence.
Pulmonary embolism is usually associated
with dyspnea along with chest pain.
(u) B. See A for explanation.
(u) C. See A for explanation.
(u) D. See A for explanation.

17.

Diagnosis/Cardiology
A 42 year-old male is brought into the ED with a complaint of chest pain. The pain
comes on suddenly without exertion and lasts anywhere from 10-20 minutes. The
patient has experienced this on three previous occasions. Today the patient
complains of light- headedness with the chest pain lasting longer. Vital signs T-99.3F
oral, P-106/minute and regular, R-22/minute, BP 146/86 mm Hg. EKG reveals sinus
rhythm with a rate of 100. Intervals are PR = 0.06 seconds, QRS = 0.12 seconds. A delta
wave is noted in many leads. Based upon this information what is the most likely
diagnosis?
Answers
A. Sinus tachycardia
B. Paroxysmal supraventricular tachycardia
C. Wolff-Parkinson-White syndrome
D. Ventricular tachycardia

Explanations
(u) A. See C for explanation.
(u) B. See C for explanation.
(c) C. Wolff-Parkinson-White syndrome
hallmarks on EKG include a shorten PR
interval, widened QRS, and delta waves.
Sinus tachycardia has a normal PR
interval and no delta waves. PSVT usually
has a retrograde P wave or it may be
buried in the QRS complex.
(u) D. Ventricular tachycardia has a
widened QRS as it originates in the
ventricles.

18.

Diagnosis/Cardiology
A 63 year-old male is admitted to the hospital with an exacerbation of
COPD. The electrocardiogram shows an irregularly, irregular rhythm at a
rate of 120/minute with at least three varying P wave morphologies. These
electrocardiogram findings are most suggestive of
Answers
A. atrial fibrillation.
B. multifocal atrial tachycardia.
C. atrioventricular junctional rhythm.
D. third degree heart block.

Explanations
(u) A. Atrial fibrillation is an irregularly, irregular
rhythm with no definable P waves.
(c) B. Multifocal atrial tachycardia is seen most
commonly in patients with COPD. Electrocardiogram
findings include an irregularly, irregular rhythm
with a varying PR interval and various P wave
morphologies (Three or more foci).
(u) C. Atrioventricular junctional rhythm is an escape
rhythm, because of depressed sinus node function,
with a ventricular rate between 40-60/minute.
(u) D. Third degree heart block presents with a wide
QRS at a rate less than 50/minute and blocked atrial
impulses.

19.

Health Maintenance/Cardiology
A 72 year-old female is being discharged from the hospital following an
acute anterolateral wall myocardial infarction. While in the hospital the
patient has not had any dysrhythmias or hemodynamic compromise.
Which of the following medications should be a part of her d/c meds?

Explanations
(u) A. Warfarin is not indicated since there is no role
for anticoagulation in this patient.
(c) B. ACE inhibitors have been shown to decrease
left ventricular hypertrophy and remodeling to allow
for a greater ejection fraction.
(u) C. The patient does not have any dysrhythmias
so Lanoxin is not indicated.
(u) D. The patient does not have any hemodynamic
compromise or indicators of CHF.

A. Warfarin (Coumadin)
B. Captopril (Capoten)
C. Digoxin (Lanoxin)
D. Furosemide (Lasix)
20.

Health Maintenance/Cardiology
A 44 year-old male with a known history of rheumatic fever at age 7 and
heart murmur is scheduled to undergo a routine dental cleaning. The
murmur is identified as an opening snap murmur. Patient has no known
drug allergies. What should this patient receive for antibiotic prophylaxis
prior to the dental cleaning?
Answers
A. This patient does not require antibiotic prophylaxis for a routine dental
cleaning.
B. This should receive Pen VK 250 mg p.o. QID for 10 days after the
procedure.
C. This patient should receive Amoxicillin 3.0 gms. p.o. 1 hour before the
procedure and then 1.5 gm. 6 hours after the procedure.
D. This patient should receive Erythromycin 250 mg QID for 1 day before
the procedure and then 10 days after the procedure.

Explanations
(h) A. See C for explanation.
(u) B. See C for explanation.
(c) C. These are the current recommendations from
the American Heart Association if the patient is not
allergic to penicillin.
(u) D. See C for explanation.

21.

Health Maintenance/Cardiology
A 36 year-old female presents for a refill of her oral contraceptives. She
admits to smoking one pack of cigarettes per day. She should be counseled
with regard to her risk of
Answers
A. venous thrombosis.
B. varicose veins.
C. atherosclerosis.
D. peripheral edema.

Explanations
(c) A. Women over age 35 who smoke are at
increased risk for the development of venous
thrombosis.
(u) B. Varicose veins are the result of pressure
overload on incompetent veins and not due to the
use of oral contraceptives.
(a) C. The defined risks of atherosclerosis includes
smoking, but does not include the use of oral
contraceptives.
(u) D. There is no relationship between the use of
oral contraceptives and the development of
peripheral edema.

22.

Health Maintenance/Cardiology
A 68 year-old female comes to the office for an annual physical examination. Her past
medical history is significant for a 40-pack year cigarette smoking history. She takes no
medications and has not been hospitalized for any surgery. Family medical history
reveals that her mother is living, age 87, in good health without medical problems. Her
father is deceased at age 45 from a motor vehicle crash. She has two siblings that are
alive and well. From this information, how many identifiable risk factors for
cardiovascular heart disease exist in this patient?
Answers
A. 0
B. 1
C. 2
D. 3

Explanations
(u) A. See C for explanation.
(u) B. See C for explanation.
(c) C. This patient has 2 identifiable risk
factors based upon the information
provided. These include her age 68 and
her history of cigarette smoking.
(u) D. See C for explanation.

23.

Clinical Intervention/Cardiology
Following an acute anterolateral myocardial wall infarction two days ago, a patient
suddenly develops hemodynamic deterioration without EKG changes occurring. What
complication can explain this scenario?
Answers
A. Free wall rupture
B. CVA
C. Atrial fibrillation
D. Sick sinus syndrome

Explanations
(c) A. Free wall rupture is a complication
that occurs within 72 hours of infarction.
It is seen mainly in Q wave transmural
and lateral wall infarctions.
(u) B. See A for explanation.
(u) C. See A for explanation. Atrial
fibrillation would have EKG evidence of
irregularly, irregular rate and rhythm.
(u) D. Sick sinus syndrome would have
EKG evidence of decreased rate and
loss of P waves.

24.

Clinical Intervention/Cardiology
A 48 year-old male with a history of coronary artery disease and two myocardial
infarctions complains of shortness of breath at rest and 2-pillow orthopnea. His oxygen
saturation is 85% on room air. The patient denies any prior history of symptoms. The
patient denies smoking. Results of a beta-natriuretic peptide (BNP) are elevated. What
should be your next course of action for this patient?
Answers
A. Send him home on 20 mg furosemide (Lasix) p.o. every day and recheck in one week
B. Send him home on clarithromycin (Biaxin) 500 mg p.o. BID and recheck in 1 week
C. Admit to the hospital for work up of left ventricular dysfunction
D. Admit to the hospital for work up of pneumonia

Explanations
(h) A. See C for explanation.
(h) B. See C for explanation.
(c) C. An elevated BNP is seen in a
situation where there is increased
pressure in the ventricle during
diastole. This is representative of the left
ventricle being stretched excessively
when a patient has CHF. Sending a
patient home would be inappropriate
in this case.
(u) D. See C for explanation.

25.

Clinical Intervention/Cardiology
A 48 year-old male presents to the ED with complaints of chest pressure,
dyspnea on exertion, and diaphoresis that has been present for the last
one hour. Electrocardiogram reveals normal sinus rhythm at 92/minute
along with ST segment elevation in leads V3-V5. Initial cardiac enzymes
are normal. What is the next most appropriate step in the management
of this patient?
Answers
A. Coronary artery revascularization
B. Admission for medical management
C. Administer lidocaine
D. Administer nitrates

Explanations
(c) A. The standard of care for the management of acute
ST-segment elevation MI is coronary artery
revascularization. This patient is diagnosed with an STsegment elevation MI based upon his history and EKG
findings. Cardiac enzymes are normal because of the
early presentation of this patient.
(u) B. Although this patient will be admitted to the
hospital, this patient needs to have acute management
of the myocardial infarction without delay.
(h) C. Prophylactic lidocaine has been shown to increase
morbidity and mortality from acute MI when used in
this setting.
(u) D. Although pain control is a goal for patients with
acute MI, it is not the essential medication that will
impact this patient's care to the greatest degree.

26.

Clinical Intervention/Cardiology
An unresponsive patient is brought to the ED by ambulance. He is in
ventricular tachycardia with a heart rate of 210 beats/min and a blood
pressure of 70/40 mmHg. The first step in treatment is to
Answers
A. administer IV adenosine.
B. DC cardiovert.
C. administer IV lidocaine.
D. apply overdrive pacer.

Explanations
(u) A. Adenosine is used to treat PSVT.
(c) B. The first step in treatment of unstable ventricular
tachycardia with a pulse is to cardiovert using a 100 J
countershock.
(u) C. See B for explanation.
(u) D. Overdrive pacing is indicated in Torsades de
Pointes.

27.

Clinical Therapeutics/Cardiology
Which of the following antiarrhythmic drugs can be associated with
hyper- or hypothyroidism following long-term use?
Answers
A. Quinidine
B. Amiodarone
C. Digoxin
D. Verapamil

Explanations
(u) A. See B for explanation.
(c) B. Amiodarone is structurally related to thyroxine
and contains iodine, which can induce a hyper- or
hypothyroid state.
(u) C. See B for explanation.
(u) D. See B for explanation.

28.

Clinical Therapeutics/Cardiology
Which of the following hypertensive emergency drugs has the potential
for developing cyanide toxicity?
Answers
A. Sodium nitroprusside (Nipride)
B. Diazoxide (Hyperstat)
C. Labetalol (Normodyne)
D. Alpha-methyldopa (Aldomet)

Explanations
(c) A. Sodium nitroprusside metabolization results in
cyanide ion production. It can be treated with sodium
thiosulfite, which combines with the cyanide ion to form
thiocyanate, which is nontoxic.
(u) B. See A for explanation.
(u) C. See A for explanation.
(u) D. See A for explanation.

29.

Clinical Therapeutics/Cardiology
Contraindications to beta blockade following an acute myocardial
infarction include which of the following?
Answers
A. Third degree A-V block
B. Sinus tachycardia
C. Hypertension
D. Rapid ventricular response to Atrial fibrillation/flutter

Explanations
(c) A. Beta blockade is contraindicated in second and
third heart block.
(u) B. Beta blockade has been proven to be beneficial in
sinus tachycardia, hypertension and in atrial fib/flutter
with a rapid ventricular response.
(u) C. See B for explanation.
(u) D. See B for explanation.

30.

Clinical Therapeutics/Cardiology
A 74 year-old male is diagnosed with pneumonia. The physician
assistant should ensure the patient is not on which of the following
before starting therapy with clarithromycin (Biaxin)?
Answers
A. Lisinopril (Zestril)
B. Furosemide (Lasix)
C. Simvastatin (Zocor)
D. Dipyridamole (Persantine)

Explanations
(u) A. See C for explanation.
(u) B. See C for explanation.
(c) C. Statins are known to interact with the macrolides as
they may cause prolonged QT interval, myopathy and
rhabdomyolysis.
(u) D. See C for explanation.

31.

Clinical Therapeutics/Cardiology
According to the recent JNC VII guidelines, a 34 year-old male who has
type 1 diabetes mellitus and hypertension should be started on
which type of antihypertensive agent?
Answers
A. Beta-blocker
B. Loop diuretic
C. ACE inhibitor
D. Thiazide diuretic

Explanations
(u) A. Beta blockers could potentially be harmful in a
patient with diabetes mellitus. Use a cardioselective betablocker to reduce the incidence of hypoglycemia.
(u) B. See C for explanation.
(c) C. ACE inhibitors are effective in young patients. They
are capable of providing protection to the kidney
especially in diabetes mellitus.
(u) D. See C for explanation.

32.

Clinical Therapeutics/Cardiology
Which of the following beta-adrenergic blocking agents has
cardioselectivity for primarily blocking beta-1 receptors?
Answers
A. Propranolol (Inderal)
B. Timolol (Blocadren)
C. Metoprolol (Lopressor)
D. Pindolol (Visken)

Explanations
(u) A. Propranolol and timolol are nonselective betaadrenergic antagonists.
(u) B. See A for explanation.
(c) C. Metoprolol is selective for beta-1 antagonists
(u) D. Pindolol is an antagonist with partial agonist activity.

33.

Scientific Concepts/Cardiology
Which of the following is the mechanism of action of Class III
antiarrhythmic drugs?
Answers
A. Na+ channel blocker
B. K+ channel blocker
C. Beta adrenoreceptor blocker
D. Ca++ channel blocker

Explanations
(u) A. Na+ channel blockers are Class I.
(c) B. K+ channel blockers are Class III.
(u) C. Beta adrenoreceptor blockers are Class II.
(u) D. Ca++ channel blockers are Class VI.

34.

Scientific Concepts/Cardiology
In congestive heart failure the mechanism responsible for the
production of an S3 gallop is
Answers
A. contraction of atria in late diastole against a stiffened ventricle.
B. rapid ventricular filling during early diastole.
C. vibration of a partially closed mitral valve during mid to late
diastole.
D. secondary to closure of the mitral valve leaflets during systole.

Explanations
(u) A. Atrial contraction against a noncompliant ventricle is
the mechanism responsible for S4.
(c) B. Rapid ventricular filling during early diastole is the
mechanism responsible for the S3.
(u) C. Vibration of a partially closed mitral valve during mid
to late diastole is the mechanism responsible for the
Austin-Flint murmur of aortic regurgitation.
(u) D. Closure of the mitral valve leaflets during systole is
the mechanism responsible for part of the S1 heart sound.

35.

Scientific Concepts/Cardiology
What is the most likely mechanism responsible for retinal
hemorrhages and neurologic complications in a patient with
infective endocarditis?
Answers
A. Metabolic acidosis
B. Systemic arterial embolization of vegetations
C. Hypotension and tachycardia
D. Activation of the immune system

Explanations
(u) A. See B for explanation.
(c) B. The vegetations that occur during infective
endocarditis can become emboli and can be dispersed
throughout the arterial system.
(u) C. See B for explanation.
(u) D. Glomerulonephritis and arthritis result from
activation of the immune system.

36.

Scientific Concepts/Cardiology
During an inferior wall myocardial infarction the signs and symptoms
of nausea and vomiting, weakness and sinus bradycardia are a result
of what mechanism?
Answers
A. Increased sympathetic tone
B. Increased vagal tone
C. Activation of the renin-angiotensin system
D. Activation of the inflammatory and complement cascade system

Explanations
(u) A. See B for explanation.
(c) B. Increased vagal tone is common in inferior wall MI; if
the SA node is involved, bradycardia may develop.
(u) C. See B for explanation.
(u) D. See B for explanation.

37.

Scientific Concepts/Cardiology
Which of the following is the most common cause of secondary
hypertension?
A. Renal parenchymal disease
B. Primary aldosteronism
C. Oral contraceptive use
D. Cushing's syndrome

Explanations
(c) A. Renal parenchymal disease is the most common
cause of secondary hypertension.
(u) B. Primary aldosteronism can cause secondary
hypertension, but it is not the most common cause.
(u) C. Oral contraceptives can cause small increases in
blood pressure but considerable increases are much less
common.
(u) D. Cushing's disease is a less common cause of
secondary hypertension.

38.

Clinical Therapeutics/Cardiology
Which of the following medication classes is the treatment of choice
in a patient with variant or Prinzmetal's angina?
A. Calcium channel blockers
B. ACE inhibitors
C. Beta blockers
D. Angiotensin II receptor blockers

Explanations
(c) A. Calcium channel blockers are effective
prophylactically to treat coronary vasospasm associated
with variant or Prinzmetal's angina.
(u) B. ACE inhibitors are not a treatment for coronary
vasospasm.
(h) C. Beta blockers have been noted to exacerbate
coronary vasospasm potentially leading to worsening
ischemia.
(u) D. Angiotensin II receptor blockers are not a treatment
for coronary vasospasm.

39.

Clinical Therapeutics/Cardiology
A 63 year-old female with history of diabetes mellitus presents for blood
pressure follow-up. At her last two visits her blood pressure was 150/92
and 152/96. Today in the office her blood pressure is 146/92. Recent
blood work shows a Sodium 140 mEq/L, Potassium 4.2 mEq/L, BUN of 23
mg/dL, and Creatinine of 1.1 mg/dL. Which of the following is the most
appropriate initial medication in this patient?
A. Terazosin (Hytrin)
B. Atenolol (Tenormin)
C. Lisinopril (Zestril)
D. Hydrochlorothiazide (HCTZ)

Explanations
(u) A. Alpha blockers are not the treatment of choice in a
diabetic with hypertension.
(u) B. Patients with hypertension and diabetes may
require a Beta blocker, but it should be added to an
ACE inhibitor
if the ACE inhibitor is ineffective on its own.
(c) C. ACE inhibitors should be part of the initial
treatment of hypertension in diabetics because of
beneficial effects in
diabetic nephropathy and is the most appropriate initial
medication.
(u) D. Patients with hypertension and diabetes mellitus
may require a diuretic, but it should be added to an
ACE
inhibitor if the ACE inhibitor is ineffective on its own.

40.

Diagnostic Studies/Cardiology
What is the EKG manifestation of cardiac end-organ damage due to
hypertension?
A. Right bundle branch block
B. Left ventricular hypertrophy
C. Right ventricular hypertrophy
D. ST segment elevation in lateral precordial leads

Explanations
(u) A. Right bundle branch block is caused by a delay in
the conduction system in the right ventricle. It may be
caused by right ventricular hypertrophy or conditions
with higher pulmonic resistance such as cor pulmonale.
Hypertension, however, is likely to cause changes in the
left ventricle rather than the right ventricle.
(c) B. Long-standing hypertension can lead to left
ventricular hypertrophy with characteristic changes
noted on EKG.
(u)C. See A for explanation.
(u) D. ST segment elevation is a sign of acute myocardial
infarction not hypertension.

41.

Health Maintenance/Cardiology
Annual blood pressure determinations should be obtained beginning at
the age of
A. 3 years.
B. 5 years.
C. 12 years.
D. 18 years.

Explanations
(c) A. Periodic measurements of blood pressure should
be part of routine preventive health assessments
beginning at the age of 3 years.
(u) B. See A for explanation.
(u) C. See A for explanation.
(u) D. See A for explanation.

42.

History & Physical/Cardiology


Which of the following conditions would cause a positive Kussmaul's
sign on physical examination?
A. Left ventricular failure
B. Pulmonary edema
C. Coarctation of the aorta
D. Constrictive pericarditis

Explanations
(u) A. Left ventricular failure results in the back-up of
blood into the left atrium and then the pulmonary
system so it would not be associated with Kussmaul's
sign.
(u) B. Pulmonary edema primarily results in increased
pulmonary pressures rather than having effects on the
venous inflow into the heart.
(u) C. Coarctation of the aorta primarily affects outflow
from the heart due to the stenosis resulting in delayed
and decreased femoral pulses; it has no effect on
causing Kussmaul's sign.
(c) D. Kussmaul's sign is an increase rather than the
normal decrease in the CVP during inspiration. It is most
often caused by severe right-sided heart failure; it is a
frequent finding in patients with constrictive pericarditis
or right ventricular infarction.

43.

History & Physical/Cardiology


Which of the following physical findings is suggestive of atrial septal defect?
A. Fixed split S2
B. Increased pulse pressure
C. Continuous mechanical murmur
D. Difference in blood pressure between the left and right arm

Explanations
(c) A. An atrial septal defect will cause a shunt
of blood from the left to the right atrium. This
will result in an equalization in the amount of
blood entering both the left and right
ventricles which effectively eliminates the
normally wide splitting that inspiration
typically causes in hearts without an atrial
septal defect.
(u) B. Pulse pressures reflect the difference in
aortic and left ventricular volumes that occur
during ventricular systole Increased pulse
pressures are seen in aortic regurgitation
which is a different entity than atrial septal
defect.
(u) C. Continuous mechanical murmurs are
noted in patients with patent ductus
arteriosus.
(u) D. Differences in blood pressure between
the left and right arms are seen in conditions
such as coarctation of the
aorta.

44.

Clinical Therapeutics/Cardiology
A 29 year-old male presents with complaint of substernal chest pain for 12 hours.
The patient states that the pain radiates to his shoulders and is relieved with
sitting forward. The patient admits to recent upper respiratory symptoms. On
examination vital signs are BP 126/68, HR 86, RR 20, temp 100.3 degrees F. There is
no JVD noted. Heart exam reveals regular rate and rhythm with no S3 or S4. There
is a friction rub noted. Lungs are clear to auscultation. EKG shows diffuse ST
segment elevation. What is the treatment of choice in this patient?
A. Pericardiocentesis
B. Nitroglycerin
C. Percutaneous coronary intervention
D. Indomethacin (Indocin)

Explanations
(u) A. Pericardiocentesis is the treatment of
choice in a patient with a pericardial effusion
and cardiac tamponade, there is no evidence
of either of these in this patient.
(u) B. Nitroglycerin is indicated in the
treatment of chest pain related to angina.
(u) C. Percutaneous coronory intervention is
the treatment of choice in a patient with an
acute myocardial infarction.
(c) D. Indomethacin, a nonsteroidal antiinflammatory medication, is the treatment of
choice in a patient with acute
pericarditis.

45.

Diagnosis/Cardiology
A 24 year-old male presents for routine physical examination. On physical
examination, you find that the patient's upper extremity blood pressure is higher
than the blood pressure in the lower extremity. Heart exam reveals a late systolic
murmur heard best posteriorly. What is the most likely diagnosis in this patient?
A. Hypertrophic obstructive cardiomyopathy
B. Patent foramen ovale
C. Coarctation of the aorta
D. Patent ductus arteriosus

Explanations
(u) A. Patients with hypertrophic obstructive
cardiomyopathy do not present with
hypertension or weak femoral pulses.
(u) B. The murmur associated with patent
foramen ovale is a systolic ejection murmur
heard in the second and third intercostal
spaces and patients do not present with
hypertension.
(c) C. Coarctation of the aorta commonly
presents with higher systolic pressures in the
upper extremities than the lower extremities
and absent or weak femoral pulses.
(u) D. Patent ductus arteriosus is rare in
adults and patients are noted to have a
continuous rough, machinery murmur.

46.

Diagnosis/Cardiology
A 63 year-old female presents with a complaint of chest pressure for one
hour, noticed upon awakening. She admits to associated nausea,
vomiting, and shortness of breath. 12 lead EKG reveals ST segment
elevation in leads II, III, and AVF. Which of the following is the most likely
diagnosis?
A. Aortic dissection
B. Inferior wall myocardial infarction
C. Acute pericarditis
D. Pulmonary embolus

Explanations
(u) A. A patient with aortic dissection will complain of
tearing, ripping pain. EKG is often normal, but may
reveal left ventricular strain pattern.
(c) B. Myocardial infarction often presents with chest
pressure and associated nausea and vomiting. ST
segment elevation in leads II, III, and AVF are classic
findings seen in acute inferior wall myocardial
infarction.
(u) C. Acute pericarditis presents with atypical chest
pain and diffuse ST segment elevation.
(u) D. Pulmonary embolism often presents with either
no EKG changes or sinus tachycardia. Classically
described,
rarely seen findings include a large S wave in lead I, a
Q wave with T wave inversion in lead III, ST segment
depression in lead II, T wave inversion in leads V1-V4
and a transient right bundle branch block.

47.

History & Physical/Cardiology


A 12 month-old child with tetralogy of Fallot is most likely to have which
of the following clinical features?
A. Chest pain
B. Cyanosis
C. Convulsions
D. Palpitations

Explanations
(u) A. Chest pain is not a feature of tetralogy of Fallot.
(c) B. Cyanosis is very common in tetralogy of Fallot.
(u) C. Convulsions are occasionally seen as part of
severe hypoxic spells in infancy rather than a feature
of tetralogy
of Fallot.
(u) D. Palpitations are uncommon in tetralogy of
Fallot.

48.

Diagnosis/Cardiology
A 23 year-old male presents with syncope. On physical examination you
note a medium-pitched, mid-systolic murmur that decreases with
squatting and increases with straining. Which of the following is the most
likely diagnosis?
A. Hypertrophic cardiomyopathy
B. Aortic stenosis
C. Mitral regurgitation
D. Pulmonic stenosis

Explanations
(c) A. Hypertrophic cardiomyopathy is characterized
by a medium- pitched, mid-systolic murmur that
decreases with squatting and increases with
straining.
(u) B. Straining decreases the intensity of the
murmur associated with aortic stenosis and squatting
increases the intensity.
(u) C. Mitral regurgitation is characterized by a
blowing systolic murmur that radiates to the axilla, it is
not often associated with syncope.
(u) D. Pulmonic stenosis is a harsh systolic murmur
with a widely split S2, and no change with
maneuvers.

49.

Health Maintenance/Cardiology
A patient with which of the following is at highest risk for
coronary artery disease?
A. Congenital heart disease
B. Polycystic ovary syndrome
C. Acute renal failure
D. Diabetes mellitus

Explanations
(u) A. Congenital heart disease is not an established risk factor for
coronary artery disease.
(u) B. While patients with polycystic ovary syndrome have
hyperinsulimemia, they do not have the same poor
prognosis for coronary artery disease as patients with diabetes
mellitus.
(u) C. Patients with acute renal failure are not at risk for coronary
artery disease, although patients with diabetes and
chronic renal disease do have this risk.
(c) D. Patients with diabetes mellitus are in the same risk category for
coronary artery disease as those patients with
established atherosclerotic disease.

50.

Clinical Therapeutics/Cardiology
Acute rebound hypertensive episodes have been reported
to occur with the sudden withdrawal of
A. verapamil (Calan).
B. lisinopril (Prinivil).
C. clonidine (Catapres).
D. hydrochlorothiazide (HCTZ)

Explanations
(u) A. Verapamil is a calcium channel blocker and there is no
associated rebound hypertension after withdrawal.
(u) B. Lisinopril is an ACE inhibitor, which is not associated with
rebound hypertension.
(c) C. Clonidine (Catapres) is a central alpha agonist and abrupt
withdrawal may produce a rebound hypertensive
crisis.
(u) D. Hydrochlorothiazide is a thiazide diuretic, which is not
associated with rebound hypertension.

51.

Diagnosis/Cardiology
A 38 year-old female with history of coarctation of the aorta repair at the age of two presents with fevers
for four weeks. The patient states that she has felt fatigued and achy during this time. Maximum
temperature has been 102.1 degrees F. She denies cough, congestion, or other associated symptoms.
Physical examination reveals a pale tired appearing female in no acute distress. Heart reveals a new grade
III-IV/VI systolic ejection border at the apex, and a II/VI diastolic murmur at the right sternal border.
What is the most likely diagnosis?
A. Acute myocardial infarction
B. Bacterial endocarditis
C. Acute pericarditis
D. Restrictive cardiomyopathy

Explanations
(u) A. Acute MI
presents with
complaint of chest
pain, SOB, not with
fever and myalgias.
(c) B. Bacterial
endocarditis
presents as febrile
illness lasting several
days to weeks,
commonly with
nonspecific
symptoms,
echocardiogram
often reveals
vegetations on
affected valves.
(u) C. Pericarditis
does not present
with systolic or
diastolic murmur or
vegetation, more
commonly
pericardial friction
rub would be noted.
(u) D. Restrictive
cardiomyopathy will
show impaired
diastolic filling on
echocardiogram and
is not associated with
fever.

52.

Diagnostic Studies/Cardiology
A 23 year-old female with history of palpitations presents for evaluation. She
admits to acute onset of rapid heart beating lasting seconds to minutes with
associated shortness of breath and chest pain. The patient states she can
relieve her symptoms with valsalva. Which of the following is the most
appropriate diagnostic study to establish a definitive diagnosis in this patient?
A. Cardiac catheterization
B. Cardiac MRI
C. Chest CT scan
D. Electrophysiology study

Explanations
(u) A. Cardiac catheterization evaluates coronary
arteries but has no role in the diagnosis of
supraventricular tachycardia.
(u) B. Cardiac MRI cannot diagnose and define
pathway of supraventricular tachycardia.
(u) C. Chest CT scan will not establish definitive
diagnosis of supraventricular tachycardia.
(c) D. Electrophysiology study is useful in
establishing the diagnosis and pathway of
complex arrhythmias such as
supraventricular tachycardia.

53.

Clinical Therapeutics/Cardiology
Which of the following is the chief adverse effect of thiazide diuretics?
A. Hypokalemia
B. Hypernatremia
C. Hypocalcemia
D. Hypermagnesemia

Explanations
(c) A. Thiazide diuretics can induce electrolyte
changes. Principle among those is hypokalemia.
(u) B. Hyponatremia, not hypernatremia may be
a complication of thiazide diuretics.
(u) C. Thiazide diuretics cause the retention of
calcium and would not cause hypocalcemia.
(u) D. Thiazide diuretics cause the retention of
calcium and do not readily affect magnesium
levels.

54.

Clinical Intervention/Cardiology
A 25 year-old male with history of syncope presents for evaluation. The patient
admits to intermittent episodes of rapid heart beating that resolve
spontaneously. 12 Lead EKG shows delta waves and a short PR interval. Which
of the following is the treatment of choice in this patient?
A. Radiofrequency catheter ablation
B. Verapamil (Calan)
C. Percutaneous coronary intervention
D. Digoxin (Lanoxin)

Explanations
(c) A. Radiofrequency catheter ablation is the
treatment of choice on patients with accessory
pathways, such as Wolff-Parkinson-White
Syndrome.
(h) B. Calcium channel blockers such as
verapamil decrease refractoriness of the
accessory pathway or increase that of the AV
node leading to faster ventricular rates,
therefore calcium channel blockers should be
avoided in patients with WPW.
(u) C. Percutaneous coronary intervention is
indicated in the treatment of coronary artery
disease, not preexcitation syndromes.
(h) D. Digoxin decreases refractoriness of the
accessory pathway and increases that of the AV
node leading to faster ventricular rates. It should
therefore be avoided in patients with WPW.

55.

History & Physical/Cardiology


A patient presents for a follow-up visit for chronic hypertension. Which of the
following findings may be noted on the fundoscopic examination of this
patient?
A. cherry-red fovea
B. boxcar segmentation of retinal veins
C. papilledema
D. arteriovenous nicking

Explanations
(u) A. Cherry-red fovea and boxcar
segmentation of the retinal veins are findings
seen in central retinal artery occlusion.
(u) B. See letter A for explanation.
(u) C. Papilledema is noted in conditions causing
increased intracranial pressure.
(c) D. Arteriovenous nicking is common in chronic
hypertension.

56.

Diagnostic Studies/Cardiology
Which of the following diagnostic tests should be ordered initially to evaluate for
suspected deep venous thrombosis of the leg?
A. Venogram
B. Arteriogram
C. Duplex ultrasound
D. Impedance plethysmography

Explanations
(u) A. Venogram has been replaced by
noninvasive tests due to discomfort, cost,
technical difficulties, and complications,
such as phlebitis.
56
(h) B. Thrombophlebitis is a venous
problem, not an arterial one. Any
unnecessary invasive procedure is
potentially harmful.
(c) C. Ultrasound is the technique of
choice to detect deep venous thrombosis
in the leg.
(a) D. Impedance plethysmography is
equivalent to ultrasound in detecting
thrombi of the femoral and popliteal
veins,
but it may miss early, nonocclusive
thrombi.

57.

Diagnosis/Cardiology
A 36 year-old patient with cardiomyopathy secondary to viral myocarditis develops
fatigue, increasing dyspnea, and lower extremity edema over the past 3 days. He
denies fever. A chest x-ray shows no significant increase in heart size, but reveals
prominence of the superior pulmonary vessels. Based on these clinical findings, which
of the following is the most likely diagnosis?
A. Heart failure
B. Subacute bacterial endocarditis
C. Pulmonary embolus
D. Pneumonia

Explanations
(c) A. Given the presence of
cardiomyopathy, the patient's heart has
decreased functional reserve. The
symptoms and chest x-ray findings are
typical of congestive heart failure.
(u) B. Endocarditis occurs as a result of
infection that primarily occurs in the
blood stream. Endocarditis would
present with signs of infection or seeding
rather than signs of heart failure.
(u) C. Pulmonary embolus usually
presents with an acute onset of chest
pain, severe dyspnea, and anxiety.
(u) D. Pneumonia is less likely since there
is no fever and edema is not usually
associated with pneumonia.

58.

Clinical Intervention/Cardiology
Which of the following is first-line treatment for symptomatic bradyarrhythmias due
to sick sinus syndrome (SSS)?
A. Permanent pacemaker
B. Radiofrequency ablation
C. Antiarrhythmics
D. Anticoagulation therapy

Explanations
(c) A. Permanent pacemakers are the
therapy of choice in patients with
symptomatic bradyarrhythmias in sick
sinus syndrome.
(u) B. Radiofrequency ablation is used
for the treatment of accessory pathways
in the heart. (u) C. See A for explanation.
(u) D. See A for explanation.

59.

History & Physical/Cardiology


What type of chest pain is most commonly associated with a dissecting aortic aneurysm?
A. Squeezing
B. Dull, aching
C. Ripping, tearing
D. Burning

Explanations
(u) A. Squeezing pain is more
characteristic of angina or
esophageal pain.
(u) B. Dull, aching pain is more
characteristic of chest wall pain,
possibly angina, or anxiety.
(c) C. A dissecting aortic aneurysm
often presents with a very severe
ripping, tearing-like pain.
(u) D. Burning pain is more
characteristic of esophageal reflux,
esophagitis, or tracheobronchitis.

60.

Health Maintenance/Cardiology
A 52 year-old obese female with a history of hypertension, tobacco abuse, and
hyperlipidemia presents for routine follow-up. Which of her risk factors for coronary
atherosclerosis is not modifiable?
A. Age
B. High LDL
C. Hypertension
D. Obesity

Explanations
(c) A. Age is a non modifiable risk
factor, as is family history of
premature coronary heart disease
(u) B. High LDL is a modifiable risk
factor, as is Hypertension, low HDL,
obesity, tobacco abuse, physical
inactivity
(u) C. See B for explanation.
(u) D. See B for explanation.

61.

Diagnosis/Cardiology
An 8 year-old boy is brought to a health care provider complaining of dyspnea and fatigue.
On physical examination, a continuous machinery murmur is heard best in the second left
intercostal space and is widely transmitted over the precordium. The most likely diagnosis
is
A. ventricular septal defect.
B. atrial septal defect.
C. congenital aortic stenosis.
D. patent ductus arteriosus.

Explanations
(u) A. Ventricular septal defect
causes a holosystolic murmur
rather than a continuous
machinery-like murmur.
(u) B. Atrial septal defect causes a
fixed split S2 rather than a
continuous systolic heart murmur.
(u) C. Congenital aortic stenosis
causes a crescendo-decrescendo
systolic murmur heard best in the
second
intercostal space.
(c) D. Patent ductus arteriosus is
classically described in children as a
continuous machinery-type
murmur that is
widely transmitted across the
precordium.

62.

Diagnostic Studies/Cardiology
A 63 year-old male with history of hypertension and tobacco abuse presents
complaining of dyspnea on exertion for two weeks. The patient admits to one
episode of chest discomfort while shoveling snow which was relieved after five
minutes of rest. Vital signs are BP 130/70, HR 68, RR 14. Heart exam reveals regular rate
and rhythm, normal S1 and S2, no murmur, gallop, or rub. Lungs are clear to
auscultation bilaterally. There is no edema noted. Which of the following is the most
appropriate initial diagnostic study for this patient?
A. Helical CT scan
B. Chest x-ray
C. Nuclear stress test
D. Cardiac catheterization

Explanations
(u) A. Helical CT scan aids in the diagnosis
of pulmonary embolism, not in the
evaluation of angina.
(u) B. Chest x-ray is not used as a
diagnostic study to evaluate symptoms of
angina or coronary heart disease.
(c) C. In patients with classic symptoms of
angina, nuclear stress testing is the most
widely used test for diagnosis of
ischemic heart disease.
(u) D. Coronary angiography is indicated
in patients with classic stable angina who
are severely symptomatic despite
medical therapy and are being
considered for percutaneous
intervention (PCI), patients with
troublesome symptoms that are difficult
to diagnose, angina symptoms in a
patient who has survived sudden cardiac
death event, patients with ischemia on
noninvasive testings.

63.

Clinical Intervention/Cardiology
A 52 year-old male with history of hypertension and hyperlipidemia presents with an
acute myocardial infarction. Urgent cardiac catheterization is performed and shows
a 90% occlusion of the left anterior descending artery. The other arteries have
minimal disease. Ejection fraction is 45%. Which of the following is the treatment of
choice in this patient?
A. Coronary artery bypass grafting (CABG)
B. Streptokinase
C. Percutaneous coronary intervention (PCI)
D. Warfarin (Coumadin)

Explanations
(u) A. Percutaneous coronary
intervention is a better, less invasive
alternative to CABG for single vessel
coronary artery disease.
(h) B. Streptokinase is not commonly used
for treatment of acute myocardial
infarction because it is ineffective at
opening the occluded artery and
reducing mortality. Streptokinase would
be harmful because it would increase the
risk of bleeding.
(c) C. Immediate coronary angiography
and primary percutaneous coronary
intervention have been shown to be
superior to thrombolysis.
(u) D. Warfarin is used to prevent
thrombosis and not for acute treatment.

64.

Clinical Intervention/Cardiology
A patient presents with an acutely painful and cold left leg. Distal pulses are absent.
Leg is cyanotic. There are no signs of gangrene or other open lesions. Symptoms
occurred one hour ago. Which of the following treatments is most appropriate?
A. Vena cava filter
B. Embolectomy
C. Amputation
D. Aspirin

Explanations
(u) A. Vena cava filters are used in the
management of venous thromboembolic
disease when anticoagulation cannot be
done.
(c) B. Embolectomy within 4 to 6 hours is
the treatment of choice.
(h) C. Amputation is done only when no
viable tissue is present. Cutting off a
viable limb is never a good idea.
(u) D. Aspirin is used in the prevention
and treatment of coronary disease and
has no role in the treatment of
peripheral arterial embolism.

65.

Clinical Therapeutics/Cardiology
Which of the following medications used in the treatment of
supraventricular tachycardia is able to cause sinus arrest and
asystole for a few seconds while it breaks the paroxysmal
supraventricular tachycardia?
A. Digoxin (Lanoxin)
B. Adenosine (Adenocard)
C. Verapamil (Calan)
D. Quinidine (Quinaglute)

Explanations
(u) A. Digoxin is not used for the acute termination of
supraventricular tachycardia.
(c) B. Adenosine is an endogenous nucleoside that results in
profound (although transient) slowing of the AV
conduction and sinus node discharge rate. This agent has a very
short half-life of 6 seconds.
(u) C. Although verapamil may be used for the termination of
acute supraventricular tachycardia, it does not lead to
sinus arrest in therapeutic doses.
(u) D. Quinidine is rarely used today and is not indicated for the
termination of supraventricular tachycardia.

66.

Diagnosis/Cardiology
An elderly female presents for evaluation of exertional
syncope, dyspnea, and angina. She admits that previous to
these symptoms she had insidious progression of fatigue that
caused her to curtail her activities. Which of the following is
the most likely diagnosis?
A. Aortic stenosis
B. Aortic regurgitation
C. Mitral stenosis
D. Mitral valve prolapse

Explanations
(c) A. The major symptoms of aortic stenosis are exertional syncope,
dyspnea, and angina. Symptoms do not become apparent for a
number of years and usually are not present until the valve is
narrowed to less than 0.5 cm to 2 cm of valve surface area.
(u) B. Patients with aortic regurgitation are likely to complain of an
uncomfortable awareness of their heart, especially when lying
down. These patients develop sinus tachycardia with exertion and
complain of palpitations and head pounding with activity.
(u) C. The symptoms related to mitral stenosis are related to
increased pulmonary pressure after the left atrium can no longer
overcome the outflow obstruction.
(u) D. Patients with mitral valve prolapse are typically
asymptomatic throughout their lives, although a wide range of
symptoms is possible. When symptoms do occur, palpitations from
arrhythmias are most common along with lightheadedness.
Syncope is not part of this disease process.

67.

History & Physical/Cardiology


Which of the following would you expect on physical
examination in a patient with mitral valve stenosis?
A. Systolic blowing murmur
B. Opening snap
C. Mid-systolic click
D. Paradoxically split S2

Explanations
(u) A. Mitral stenosis is a diastolic, not a systolic murmur.
(c) B. Mitral stenosis is characterized by a mid-diastolic opening
snap.
(u) C. Mid-systolic clicks are noted in mitral valve prolapse, not
mitral stenosis.
(u) D. Paradoxical splitting of S2 occurs in aortic stenosis not mitral
stenosis.

68.

Scientific Concepts/Cardiology
Which of the following is the most common cause for acute
myocardial infarction?
A. Occlusion caused by coronary microemboli
B. Thrombus development at a site of vascular injury
C. Congenital abnormalities
D. Severe coronary artery spasm

Explanations
(u) A. Coronary microemboli occlusion is a rare cause of acute
myocardial infarction.
(c) B. Acute myocardial infarction occurs when a coronary artery
thrombus develops rapidly at a site of vascular
injury. In most cases, infarction occurs when an atherosclerotic
plaque fissures, ruptures, or ulcerates and when conditions favor
thrombogenesis, so that a mural thrombus forms at the site of
rupture and leads to coronary artery occlusion.
(u) C. Congenital abnormalities are rare causes of acute MI.
(u) D. Severe coronary artery spasm is more likely to result in
Prinzmetal's angina rather than true infarction.

69.

Health Maintenance/Cardiology
A 78 year-old male with history of coronary artery disease status post CABG and ischemic cardiomyopathy
presents with complaint of progressive dyspnea and orthopnea. He also complains of lower extremity
edema. The patient denies fever, chest pain, or cough. On physical examination, vital signs are BP 120/68,
HR 75 and regular, RR 22, afebrile. You note the patient to have an S3 heart sound, jugular venous
distention, and 2+ lower extremity edema. The patient is admitted and treated. Upon discharge from the
hospital, the patient should be educated to monitor which of the following at home?
A. Daily weights
B. Daily spirometry
C. Daily blood glucose
D. Daily fat intake

Explanations
(c) A. Home
monitoring of daily
weights can alert
the health care
provider to the
early recognition of
worsening heart
failure.
(u) B. Spirometry
monitoring is
important in a
patient with
asthma, not heart
failure.
(u) C. Daily blood
glucose monitoring
is important in a
patient with
diabetes, not heart
failure.
(u) D. Daily fat
intake is important,
but will not improve
his heart failure
management.

70.

Scientific Concepts/Cardiology
Which of the following is the most common cause of arterial embolization?
A. Rheumatic heart disease
B. Myxoma
C. Atrial fibrillation
D. Venous thrombosis

Explanations
(u) A. Rheumatic
heart disease is a
rare cause of
embolization
(u) B. Myxoma is a
rare cause of
embolization.
(c) C. Atrial
fibrillation is
present in 60-70%
of patients with
arterial emboli and
is associated with
left atrial
appendage
thrombus.
(u) D. Venous
thrombosis may be
a cause of
embolization
paradoxically, but is
uncommon.

71.

Scientific Concepts/Cardiology
The most common arrhythmia encountered in patients with mitral stenosis is
A. atrial flutter.
B. atrial fibrillation.
C. paroxysmal atrial tachycardia.
D. atrio-ventricular dissociation.

Explanations
(u) A. See B for explanation.
(c) B. Mitral stenosis leads to enlargement of
the left atrium, which is the major
predisposing risk factor for the
development of atrial fibrillation.
(u) C. See B for explanation.
(u) D. See B for explanation.

72.

Clinical Therapeutics/Cardiology
Long term use of which of the following drugs may cause a drug-induced lupustype eruption?
A. prednisone
B. tetracycline
C. procainamide
D. oral contraceptives

Explanations
(u) A. Prednisone is not implicated in druginduced skin reactions.
(u) B. Tetracycline and sulfonamides are
known to cause a photosensitive rash on sun
exposed areas of the skin.
(c) C. Procainamide and hydralazine are the
most common drugs that may cause a lupuslike eruption.
(u) D. Oral contraceptives may induce
erythema nodosum.

73.

Scientific Concepts/Cardiology
Which of the following is a cause of high output heart failure?
A. myocardial ischemia
B. complete heart block
C. aortic stenosis
D. thyrotoxicosis

Explanations
(u) A. Low output heart failure occurs
secondary to ischemic heart disease,
hypertension, dilated cardiomyopathy,
valvular and pericardial disease, and
arrhythmia.
(u) B. See A for explanation.
(u) C. See A for explanation.
(c) D. High output heart failure occurs in
patients with reduced systemic vascular
resistance. Examples include: thyrotoxicosis,
anemia, pregnancy, beriberi and Paget's
disease. Patients with high output heart
failure usually have normal pump function,
but it is not adequate to meet the high
metabolic demands.

74.

Diagnosis/Cardiology
A 46 year-old male with no past medical history presents complaining of chest
pain for four hours. The patient admits to feeling very poorly over the past two
weeks with fever and upper respiratory symptoms. The patient denies shortness
of breath or diaphoresis. On examination the patient appears fatigued. Vital signs
reveal a BP of 130/80, HR 90 and regular, RR 14. The patient is afebrile. Labs reveal a
Troponin I of 10.33 ug/L (0-0.4ug/L). Cardiac catheterization shows normal
coronary arteries and an ejection fraction of 40% with global hypokinesis. Which
of the following is the most likely diagnosis?
A. myocarditis
B. pericarditis
C. hypertrophic cardiomyopathy
D. coronary artery disease

Explanations
(c) A. Myocarditis often occurs secondary to
acute viral illness and causes cardiac
dysfunction. Patients will commonly have a
history of a recent febrile illness. Chest pain
may mimic that of a myocardial infarction and
Troponin I levels maybe elevated in onethird of patients. Contractile dysfunction is
seen on catheterization and/or
echocardiogram.
(u) B. Pericarditis does not typically cause
ventricular dysfunction and cardiac enzymes
are usually normal.
(u) C. Hypertrophic cardiomyopathy is
associated with ventricular hypercontractility.
(u) D. This patient had normal coronary
arteries on cardiac catheterization, no signs of
coronary artery disease.

75.

Clinical Therapeutics/Cardiology
Which of the following antihypertensive agents is considered to have both
alpha- and beta-blocker activities?
A. carvedilol (Coreg)
B. hydralazine (Apresoline)
C. minoxidil (Loniten)
D. spironolactone (Aldactone)

Explanations
(c) A. Carvedilol has both alpha- and betablocker activities.
(u) B. Hydralazine and minoxidil are
considered vasodilators.
(u) C. See B for explanation.
(u) D. Spironolactone is a potassium-sparing
diuretic.

76.

Diagnosis/Cardiology
A 12 year-old boy presents to the office with pain in his legs with activity
gradually becoming worse over the past month. He is unable to ride a bicycle
with his friends due to the pain in his legs. Examination of the heart reveals an
ejection click and accentuation of the second heart sound. Femoral pulses are
weak and delayed compared to the brachial pulses. Blood pressure obtained in
both arms is elevated. Chest x-ray reveals rib notching. Which of the following is
the most likely diagnosis?
A. abdominal aortic aneurysm
B. pheochromocytoma
C. coarctation of the aorta
D. thoracic outlet syndrome

Explanations
(u) A. Abdominal aortic aneurysm is usually
asymptomatic until the patient has dissection
or rupture. It is uncommon in a child.
(u) B. Pheochromocytoma classically causes
paroxysms of hypertension due to
catecholamine release from the adrenal
medulla, but does not cause variations in blood
pressure in the upper and lower extremities.
(c) C. Coarctation is a discrete or long segment
of narrowing adjacent to the left subclavian
artery. As a result of the coarctation, systemic
collaterals develop. X-ray findings occur from
the dilated and pulsatile intercostal arteries
and the "3" is due to the coarctation site with
proximal and distal dilations.
(u) D. Thoracic outlet syndrome occurs when
the brachial plexus, subclavian artery, or
subclavian vein becomes compressed in the
region of the thoracic outlet. It is the most
common cause of acute arterial occlusion in the
upper extremity of adults under 40 years old.

77.

Clinical Therapeutics/Cardiology
According to the recent JNC VII guidelines, a 34 year-old male who has type 1
diabetes mellitus and hypertension should be started on which type of
antihypertensive agent?
A. beta-blocker
B. loop diuretic
C. ACE inhibitor
D. thiazide diuretic

Explanations
(u) A. Beta blockers could potentially be
harmful in a patient with diabetes mellitus. Use
a cardioselective beta- blocker to reduce the
incidence of hypoglycemia.
(u) B. See C for explanation.
(c) C. ACE inhibitors are effective in young
patients. They are capable of providing
protection to the kidney especially in diabetes
mellitus.
(u) D. See C for explanation.

78.

Scientific Concepts/Cardiology
A patient presents with moderate mitral stenosis. Which of the
following complications is associated with an increased risk of
systemic embolization in this patient?
A. atrial fibrillation
B. pulmonary hypertension
C. increased left atrial pressure
D. left ventricular dilatation

Explanations
(c) A. 50-80% of patients with mitral stenosis will develop
paroxysmal or chronic atrial fibrillation; 20-30% of patients with
atrial fibrillation will have systemic embolization.
(u) B. Pulmonary hypertension can occur in patients with severe
mitral stenosis with symptoms of low cardiac output and right
sided heart failure. Pulmonary hypertension does not cause
systemic embolization.
(u) C. Patients with mitral stenosis can have increased left atrial
pressures relative to the left ventricular diastolic pressures; this
does not usually cause systemic embolization.
(u) D. Left ventricular dilatation is more common in aortic valve
disease than mitral valve disease.

79.

Diagnostic Studies/Cardiology
A 19 year-old female presents with complaint of palpitations.
On examination you note the patient to have particularly long
arms and fingers and a pectus excavatum. She has a history of
joint dislocation and a recent ophthalmologic examination
revealed ectopic lentis. Which of the following echocardiogram
findings would be most consistent with this patient's physical
features?
A. right atrial enlargement B. aortic root dilation
C. pulmonic stenosis
D. ventricular septal defect

Explanations
(u) A. Patients with Marfan's syndrome commonly have mitral
valve prolapse and possibly aortic regurgitation. Right atrial
enlargement, pulmonic stenosis and ventricular septal defect are
not commonly seen.
(c) B. This patient has the signs and symptoms consistent with
Marfan's syndrome. Ectopia lentis, aortic root dilation and aortic
dissection are major criteria for the diagnosis of the disease.
(u) C. See A for explanation. (u) D. See A for explanation.

80.

Diagnosis/Cardiology
A patient presents with chest pain. ECG done in the emergency
department reveals ST segment elevation in leads II, III, and
AVF. This is most consistent with a myocardial infarction in
which of the following areas?
A. anterior wall
B. inferior wall
C. posterior wall
D. lateral wall

Explanations
(u) A. Anterior wall myocardial infarction is characterized by ST
segment elevation in 1 or more of the precordial (V1- V6) leads.
(c) B. Inferior wall myocardial infarction is characterized by ST
segment elevation in leads II, III, and AVF.
(u) C. Posterior wall myocardial infarction is characterized by ST
segment depression in leads V1-V3 and a large R wave in leads
V1-V3.
(u) D. Lateral wall myocardial infarction is characterized by ST
segment elevation in leads I and AVL.

81.

Clinical Therapeutics/Cardiology
Which of the following is an absolute contraindication to
thrombolytic therapy in a patient with an acute ST segment
elevation myocardial infarction?
A. history of severe hypertension presently controlled B.
current use of anticoagulation therapy
C. previous hemorrhagic stroke
D. active peptic ulcer disease

Explanations
(u) A. See C for explanation.
(u) B. See C for explanation.
(c) C. Absolute contraindications to thrombolytic therapy include
a previous hemorrhagic stroke, a stroke within one year, a
known intracranial neoplasm, active internal bleeding, and a
suspected aortic dissection. Severe, but controlled hypertension,
use of anticoagulation, and active peptic ulcer disease are relative
contraindications in which the risk/benefit ratio must be weighed
in each patient.
(u) D. See C for explanation.

82.

Health Maintenance/Cardiology
A postmenopausal woman is at greatest risk of death from
which of the following?
A. stroke
B. heart disease
C. ovarian cancer
D. breast cancer

Explanations
(u) A. See B for explanation.
(c) B. Although women tend to be concerned about dying from
breast cancer, heart disease is the number one killer of
postmenopausal women.
(u) C. See B for explanation.
(u) D. See B for explanation.

83.

Diagnosis/Cardiology
A 46 year-old female is being evaluated for a new-onset hypertension that was
discovered on screening at her workplace. The patient had several readings
revealing systolic and diastolic hypertension. Patient is currently on no
medications. Physical examination is unremarkable. A complete laboratory
evaluation revealed hypokalemia as the only abnormality. Which of the
following is the most likely diagnosis for this patient?
A. pheochromocytoma
B. renal artery stenosis
C. coarctation of the aorta
D. primary aldosteronism

Explanations
(u) A. Pheochromocytoma will result in an
increase in the production and release of
catecholamines, which results in an increase in
urinary metanephrines on testing.
(u) B. Renal artery stenosis is identified by an
abnormal radionuclide uptake on the affected
kidney.
(u) C. Coarctation of the aorta is identified by
delayed and weakened femoral pulses along
with a blood pressure in the lower extremities
significantly lower than in the upper extremities.
(c) D. Primary aldosteronism has an increased
aldosterone secretion, which causes the
retention of sodium and the loss of potassium.
This should be the primary consideration for this
patient.

84.

Clinical Intervention/Cardiology
A 54 year-old female who has diabetes presents with rubor, absence of hair,
and brittle nails of her left foot. She complains of leg pain that awakens her at
night. Examination reveals a femoral bruit with diminished popliteal and
pedal pulses on the left side. The most appropriate therapy would be
A. vasodilator therapy.
B. bypass surgery.
C. exercise program.
D. embolectomy.

Explanations
(u) A. Vasodilator therapy is not indicated.
(c) B. Bypass surgery is indicated in the presence
of rest pain and provides relief of symptoms in
80 to 90% of patients.
(u) C. While an exercise program is appropriate
with claudication, rest pain is a surgical
indication.
(u) D. Embolectomy is used for acute arterial
occlusion.

85.

Clinical Therapeutics/Cardiology
Which electrolyte abnormality is associated with an increase in the risk for
digoxin toxicity?]
A. hypercalcemia
B. hypokalemia
C. hypermagnesemia
D. hyponatremia

Explanations
(u) A. See B for explanation.
(c) B. Decreased concentration of potassium
results in the increased activity of cardiac
glycosides by increasing tissue binding and
decreasing renal excretion of digoxin. Potassium
loss is the only significant electrolyte abnormality
that significantly affects digoxin metabolism.
(u) C. See B for explanation.
(u) D. See B for explanation.

86.

Health Maintenance/Cardiology
A 56 year-old male, status post myocardial infarction, is noted to have left
ventricular hypertrophy and an ejection fraction of 38%. Which of the
following medications should be prescribed to prevent the development of
heart failure symptoms?
A. amlodipine (Norvasc)
B. furosemide (Lasix)
C. hydrochlorothiazide (HCTZ)
D. lisinopril (Zestril)

Explanations
(u) A. See D for explanation.
(u) B. See D for explanation.
(u) C. See D for explanation.
(c) D. ACE inhibitors have been shown to
markedly improve survival and are also
recommended for prevention of symptoms in
patients at risk for heart failure.

87.

Health Maintenance/Cardiology
A 74 year-old patient presents with signs and symptoms of heart failure. EKG shows the
patient to be in atrial fibrillation at a rate of 80 bpm. Blood pressure is 120/76. The
patient denies complaint of palpitations, chest pain, or syncope. Which of the
following is the most important long term therapy in this patient?
A. verapamil (Calan)
B. amiodarone (Cordarone)
C. furosemide (Lasix)
D. warfarin (Coumadin)

Explanations
(u) A. Calcium channel blockers are
utilized in rate control of atrial
fibrillation. This patient's rate is
controlled at 80bpm presently.
(u) B. Antiarrhythmic therapy may be
considered in patients with atrial
fibrillation; however anticoagulation
therapy must occur first.
(u) C. Diuretics may be indicated in the
acute treatment of heart failure;
however they may not be needed long
term.
(c) D. Patients with atrial fibrillation have
an increased risk for stroke, therefore
these patients need anticoagulation
with warfarin to an INR of 2.0-3.0.

88.

Diagnostic Studies/Cardiology
Which of the following ECG findings is consistent with hyperkalemia?
A. prolonged QT interval
B. delta wave
C. peaked T waves
D. prominent U waves

Explanations
(u) A. Prolonged QT interval is seen in
hypocalcemia.
(u) B. Delta wave is a sign of ventricular
preexcitation seen in Wolf-ParkinsonWhite (WPW) Syndrome.
(c) C. Narrowing and peaking of T
waves are the beginning EKG changes
associated with hyperkalemia.
(u) D. Prominent U waves are a sign of
prolonged ventricular repolarization
seen in hypokalemia.

89.

History & Physical/Cardiology


A 58 year-old male presents with chest pain. Vital signs include blood pressure of
210/175, pulse 80, RR 20. Which of the following would you expect to find on physical
examination?
A. papilledema
B. carotid bruit
C. diastolic murmur
D. absent peripheral pulses

Explanations
(c) A. Malignant hypertension is
characterized by marked blood
pressure elevation with papilledema,
often with encephalopathy or
nephropathy.
(u) B. Carotid bruits are associated with
carotid artery stenosis.
(u) C. Diastolic murmurs are associated
with valvular heart disease such as
aortic regurgitation and mitral stenosis.
(u) D. Peripheral pulses are absent in
acute arterial occlusion or severe
peripheral arterial disease.

90.

Clinical Therapeutics/Cardiology
A 55 year-old diabetic female presents for a 3 month blood pressure followup. At the last visit the BP was 160/90 for the third consecutive visit. She was
placed on an ACE inhibitor and educated regarding lifestyle modifications. At
today's visit the patient complains of persistent annoying dry cough that has
been going on since the last visit. BP today is 120/70. What is the best
recommendation to control her BP?
A. add a diuretic
B. stop the ACE inhibitor and continue lifestyle modifications
C. switch patient to an Angiotensin II Receptor Blocker (ARB)
D. do nothing and recheck BP in 3 months

Explanations
(u) A. This patient's blood pressure is controlled;
there is no indication at this time to add an
additional drug.
(u) B. This patient's chronic dry cough is likely
secondary to the ACE inhibitor, the medication
should be stopped, however the patient needs
something for blood pressure control.
(c) C. This patient's chronic dry cough is likely
secondary to the ACE inhibitor, the medication
should be stopped. Angiotensin II Receptor
Blockers (ARBs) are similar to ACE inhibitors for BP
control, but do not cause cough.
(u) D. This patient's chronic dry cough is likely
secondary to the ACE inhibitor, the medication
should be stopped to encourage compliance.

91.

Diagnosis/Cardiology
A newborn is seen for an initial two week visit. Physical examination reveals a
thrill and a continuous machinery murmur in the left second intercostal
space. Which of the following is the most likely diagnosis?
A. patent ductus arteriosus
B. ventricular septal defect
C. tetralogy of Fallot
D. coarctation of the aorta

Explanations
(c) A. Patent ductus arteriosus is characterized by
a classic harsh, machinery-like murmur that is
continuous through systole and diastole. This is
heard best at the left second interspace and is
commonly associated with a thrill.
(u) B. Ventricular septal defect is characterized by
a holosystolic murmur at the lower left sternal
border.
(u) C. Tetralogy of Fallot is characterized by a
systolic thrill at the left sternal border with a
systolic ejection murmur that may or may not have
an associated systolic click.
(u) D. Coarctation of the aorta is associated with a
systolic ejection click or a short systolic murmur at
the left sternal border.

92.

History & Physical/Cardiology


A patient had an acute inferior, transmural myocardial infarction 4 days ago.
A new murmur raises the suspicion of mitral regurgitation due to papillary
muscle rupture. Which of the following murmur descriptions describes this
condition?
A. A grade III/VI diastolic murmur heard best at the apex without radiation.
B. A grade IV/VI systolic ejection murmur heard best at the base with
radiation to the left clavicle.
C. A grade II/VI systolic murmur heard best at the apex preceded by a click
and without radiation.
D. A grade IV/VI systolic murmur heard best at the apex with radiation to the
left axilla.

Explanations
(u) A. This is a classic description of mitral stenosis.
(u) B. This is a classic description for pulmonic
stenosis.
(u) C. This is a classic description for mitral valve
prolapse.
(c) D. This is a classic description of mitral
regurgitation. The papillary muscle rupture is a
complication of an acute inferior transmural
myocardial infarction, and results in a failure of
the mitral valve leaflets to close. The direction of
regurgitant flow of blood is toward the left axilla.

93.

Clinical Intervention/Cardiology
A 58 year-old male who is otherwise healthy presents with chest pain and is
found to have left main coronary artery stenosis of 75%. The most important
aspect of his management now is
A. daily aspirin to prevent MI.
B. nitrate therapy for the angina.
C. aggressive risk factor reduction.
D. referral for coronary artery revascularization.

Explanations
(u) A. See D for explanation.
(u) B. See D for explanation.
(u) C. See D for explanation.
(c) D. Although medical therapy is important,
revascularization is indicated when stenosis of the
left main coronary artery is greater than 50%.

94.

Diagnostic Studies/Cardiology
A 17 year-old woman presents to the office with recurrent episodes of
palpitations and near syncope. Initial ECG was normal. She is concerned about
these episodes since they can occur at any time. Which of the following is the
most appropriate step to pursue in her evaluation?
A. cardiac catheterization
B. tilt table testing
C. echocardiogram
D. Holter monitoring

Explanations
(u) A. A cardiac catheterization will not be
useful since the patient is at low risk for actual
coronary artery disease.
(u) B. Tilt table testing is useful only in trying to
determine vasodepressor syncope that is
related to position.
(u) C. An echocardiogram shows valves and left
ventricle function, not pathways of conduction.
(c) D. Holter monitoring will identify the heart
rhythm; an event recorder may also be useful
in this setting if the Holter monitor is not
diagnostic.

95.

Health Maintenance/Cardiology
A 37 year-old female with history of Turner's syndrome and coarctation of the
aorta repaired at the age of 3 presents for routine examination. The patient is
without complaints of chest pain, dyspnea, palpitations, or syncope. On
examination vitals signs reveal a BP of 130/76, HR 70, regular, RR 16. On cardiac
examination you note a grade II/VI systolic ejection murmur at the left sternal
border and a grade III/VI blowing diastolic murmur. Which of the following does
this patient require?
A. antibiotic prophylaxis
B. beta blocker therapy
C. chest CT
D. exercise stress test

Explanations
(c) A. This patient has a history of congenital
heart disease and presently has a murmur
consistent with aortic regurgitation. This
patient requires antibiotic prophylaxis against
infective endocarditis.
(u) B. Beta blocker therapy may increase the
amount of regurgitation because of increased
diastolic time and is not indicated in this
patient.
(u) C. This patient should undergo serial
echocardiograms, chest CT will not give
information regarding any changes in the
aortic regurgitation or ejection fraction.
(u) D. This patient is without any complaints;
exercise stress test is not indicated.

96.

Diagnostic Studies/Cardiology
A 60 year-old male with history of hypertension and hyperlipidemia presents
with intermittent chest heaviness for one month. The patient states he has had
occasional heaviness in his chest while walking on his treadmill at home or
shoveling snow. He also admits to mild dyspnea on exertion. His symptoms are
relieved with 2-3 minutes of rest. He denies lightheadedness, syncope,
orthopnea or lower extremity edema. Vitals reveal a BP of 130/90, HR 70, regular,
RR 14. Cardiac examination revealed a normal S1 and S2, without murmur or
rub. Lungs were clear to auscultation. Extremities are without edema. EKG
reveals no acute change and cardiac enzymes are negative. Which of the
following is the most appropriate next diagnostic study?
A. cardiac catheterization
B. nuclear exercise stress test
C. helical (spiral) CT
D. transthoracic Echocardiogram

Explanations
(u) A. Coronary angiography is indicated in
patients with classic stable angina who are
severely symptomatic despite medical therapy
and are being considered for percutaneous
intervention (PCI), patients with troublesome
symptoms that are difficult to diagnose, angina
symptoms in a patient who has survived
sudden cardiac death event, patients with
ischemia on noninvasive testings, a stress test is
a better initial diagnostic study for this patient.
(c) B. This patient has signs and symptoms of
classic angina; nuclear stress testing is the most
useful noninvasive procedure for diagnosis of
ischemic heart disease and evaluation of
angina in this patient.
(u) C. Helical CT is used in the diagnosis of
pulmonary embolism, not in the evaluation of
angina.
(u) D. Echocardiogram is used in the evaluation
of valvular heart disease not in the evaluation
of suspected myocardial ischemia.

97.

History & Physical/Cardiology


A 28 year-old patient presents with complaint of chest pain for two days. The patient
describes the pain as constant and sharp. It is worse with lying down, better with sitting up
and leaning forward. Vital signs are BP 120/80, HR 80, regular, RR 14 and Temperature 100.1
degrees F. Which of the following would you expect to find on physical examination?
A. lower extremity edema
B. carotid bruit
C. pericardial friction rub
D. splinter hemorrhages

Explanations
(u) A. Lower extremity edema is
seen with heart failure or
venous insufficiency, not
pericarditis.
(u) B. Carotid bruits are
associated with carotid artery
stenosis, not pericarditis.
(c) C. This patient has signs and
symptoms of pericarditis. A
pericardial friction rub is
characteristic of acute
pericarditis.
(u) D. Subungual (splinter)
hemorrhages are characteristic
of infective endocarditis, not
pericarditis.

98.

History & Physical/Cardiology


A 22 year-old male received a stab wound in the chest an hour ago. The diagnosis of pericardial
tamponade is strongly supported by the presence of
A. pulmonary edema.
B. wide pulse pressure.
C. distended neck veins.
D. an early diastolic murmur.

Explanations
(u) A. Pulmonary edema may
result with low output states as
seen with myocardial
contusions, but it is not strongly
suggestive of tamponade.
(u) B. Wide pulse pressure is
seen in conditions of high stroke
volume such as aortic
insufficiency or
hyperthyroidism. Narrow pulse
pressure is seen with cardiac
tamponade.
(c) C. Cardiac tamponade will
manifest with distended neck
veins and cold clammy skin.
(u) D. The onset of diastolic
murmur is suggestive of valvular
disease, not tamponade.

99.

Clinical Intervention/Cardiology
A 45 year-old female presents with complaint of lower extremity discomfort. The patient
admits to dull aching of the left lower extremity. The discomfort is worse after standing for
long periods of time. Examination reveals dilated, tortuous and elongated veins on the medial
aspect of the left leg. Pedal pulses are +2/4 bilaterally. There are no skin changes or lower
extremity edema noted. Which of the following is the most appropriate initial treatment of
choice in this patient?
A. heparin
B. compression stockings
C. furosemide (Lasix)
D. thrombectomy

Explanations
(u) A. Heparin is used in the
treatment of deep vein
thrombosis not varicose veins.
(c) B. This patient has signs and
symptoms of varicose veins.
Initial treatment with
compression stockings may
prolong or avoid the need for
surgery.
(u) C. This patient has no signs of
edema or venous insufficiency
requiring diuretic therapy.
(u) D. Thrombectomy is
indicated in a patient with an
arterial thrombus, this patient
has intact pulses and no pallor.

100.

Diagnosis/Cardiology
A 55 year-old male presents with complaint of sudden ripping chest
pain that radiates into the abdomen. On examination the patient is
found to have diminished peripheral pulses and a diastolic murmur.
EKG reveals left ventricular hypertrophy. Which of the following is
the most likely diagnosis?
A. acute myocardial infarction
B. pulmonary embolism
C. acute pericarditis
D. aortic dissection

Explanations
(u) A. Pain associated with a myocardial infarction is
commonly a retrosternal pressure, squeezing, or
heaviness. ST segment elevation on EKG would be
expected.
(u) B. A pulmonary embolism is associated with
retrosternal pain; however chest pain is not always
present. Patients more commonly will have a sudden onset
of dyspnea. PE is not usually associated with a diastolic
murmur or diminished pulses.
(u) C. Acute pericarditis is characterized by sharp, knifelike pain that is worse with lying supine and better with
sitting up and leaning forward. One would expect to find a
pericardial friction rub on auscultation.
(c) D. Aortic dissection is characterized by a ripping or
tearing type pain with radiation to the neck, back or
abdomen. Left ventricular hypertrophy is often seen on
EKG secondary to longstanding hypertension. A diastolic
murmur is often present secondary to aortic insufficiency.

101.

History & Physical/Cardiology


A 78 year-old patient who is in acute distress with near-syncope and
lightheadedness is being examined. The patient's pulse is 40/min and
blood pressure is 90/56 mm Hg. Examination of the patient at 45
degrees of elevation reveals cannon "a" waves. Which of the
following is the most likely explanation for these abnormal waves?
A. atrioventricular dissociation
B. aortic stenosis
C. systolic hypertension
D. left ventricular hypertrophy

Explanations
(c) A. The patient is in a third-degree heart block with the
atria contracting against a closed atrioventricular valve,
which would be the scenario in a patient who has an
escape rate of 40. Elderly patients are at risk for heart
conduction problems such as complete heart block.
(u) B. Left ventricular hypertrophy, systolic hypertension,
and aortic stenosis are not causes of "a" waves.
(u) C. See B for explanation.
(u) D. See B for explanation.

102.

Scientific Concepts/Cardiology
Which of the following occurs as a result of pulmonary
hypertension?
A. left atrial enlargement
B. aortic stenosis
C. coronary artery spasm
D. right ventricular enlargement

Explanations
(u) A. See D for explanation.
(u) B. See D for explanation.
(u) C. See D for explanation.
(c) D. In pulmonary hypertension increased resistance
within pulmonary circulation causes the right ventricle to
work harder and eventually enlarge in response. Other
changes that may occur are right atrial enlargement,
decreased left ventricular cavity size, and tricuspid
regurgitation.

103.

Diagnostic Studies/Cardiology
A 56 year-old male presents to the office with a history of abdominal
aortic aneurysm. He was told that he will need ongoing evaluation to
assess whether the aneurysm is expanding. What is the
recommended study to utilize in this situation?
A. plain film of the abdomen
B. serial abdominal exam
C. ultrasound of the abdomen
D. angiography of the abdominal aorta

Explanations
(u) A. Although some abdominal aortic aneurysms are
calcified, abdominal radiography may demonstrate the
calcified outline of the aneurysm. However, about 25% of
aneurysms are not calcified and cannot be visualized by
plain x-ray.
(u) B. Serial abdominal exams are not sensitive in
detecting progression of abdominal aortic aneurysms.
(c) C. An abdominal ultrasound can delineate the
transverse and longitudinal dimensions of an abdominal
aortic aneurysm and may detect mural thrombus.
Abdominal ultrasound is best used to screen patients at
risk for the development of this condition.
(u) D. Contrast aortography is used commonly for the
evaluation of patients with aneurysms before surgery,
but it has no role in the serial assessment of patients being
followed on a chronic basis.

104.

Diagnosis/Cardiology
A 55 year-old morbidly obese male is seen in the office for routine
examination. He has a history of pulmonary hypertension and cor
pulmonale. Examination reveals a visible jugular venous pulse and a
systolic flow murmur on the right side of the sternum. Which of the
following is the most likely diagnosis?
A. mitral insufficiency
B. tricuspid insufficiency
C. hepatic vein thrombosis
D. aneurysm of the thoracic aorta

Explanations
(u) A. Mitral insufficiency results in the accumulation of
blood primarily in the pulmonary system and not the
right side of the heart.
(c) B. Tricuspid insufficiency will result in blood being put
back into the right side of the body with increased jugular
pulsation in the neck, along with a palpable venous pulse
in the liver.
(u) C. Hepatic vein thrombosis or Budd-Chiari syndrome
is associated with cirrhosis and liver clotting abnormalities
and is not due to right-sided heart failure.
(u) D. Thoracic aorta aneurysm results in a widened
mediastinum that is fairly asymptomatic until it results in
rupture or dissection. These are typically found as
incidental findings unless they are symptomatic from
dissection or rupture, which causes severe chest pain or a
severe tearing sensation in the chest.

105.

Clinical Intervention/Cardiology
A 68 year-old patient presents after a syncopal episode. The patient
has a history of coronary artery disease and ischemic
cardiomyopathy. Echocardiogram shows an ejection fraction of 20%.
Electrophysiology study reveals inducible sustained ventricular
tachycardia from the left ventricle. Which of the following is the
most appropriate therapy in this patient?
A. implantable defibrillator
B. metoprolol (Lopressor)
C. radiofrequency ablation
D. warfarin (Coumadin)

Explanations
(c) A. Patients with symptomatic ventricular tachycardia
(VT) or sustained VT and left ventricular dysfunction are at
increased risk for sudden cardiac death. An implantable
defibrillator is the treatment of choice.
(u) B. Beta blockers are used in patients with
nonsustained VT and normal ventricular function. They
may be used as an adjunct to, but not in place of,
implantable defibrillator therapy in patients with
symptomatic VT or sustained VT.
(u) C. Radiofrequency ablation is indicated in patients
with outflow tract or fascicular tachycardia, not left sided
VT.
(u) D. Anticoagulation therapy is indicated in patients with
atrial fibrillation not VT.

106.

Scientific Concepts/Cardiology
Which of the following is the most common cause of infective endocarditis in an IV
drug abuser?
A. Haemophilus parainfluenza
B. Enterococci
C. Staphylococcus aureus
D. Viridans streptococci

Explanations
(u) A. See C for explanation.
(u) B. See C for explanation.
(c) C. S. aureus accounts for more than
60% of all cases of endocarditis in IV
drug abusers.
(u) D. See C for explanation.

107.

History & Physical/Cardiology


Which of the following is an expected finding in a patient with a diagnosis of an arterial
embolism?
A. lower extremity edema
B. stasis dermatitis
C. palpable cord
D. pulselessness

Explanations
(u) A. Lower extremity edema is
commonly associated with venous
insufficiency, not arterial embolism.
(u) B. Stasis dermatitis is commonly
seen in patients with venous
insufficiency, not arterial embolism.
(u) C. A palpable cord is commonly
found in a patient with a DVT, not
arterial embolism.
(c) D. Pulselessness is a sign of acute
ischemia secondary to arterial
embolism.

108.

Scientific Concepts/Cardiology
Which of the following factors in patients with chronic venous insufficiency
predisposes them to development of skin ulcers?
A. Increased intravascular oncotic pressure
B. Leakage of fibrinogen and growth factors into the interstitial space
C. Decreased capillary leakage
D. Inherited deficiency of protein C

Explanations
(u) A. Decreased intravascular oncotic
pressure can cause swelling
(c) B. Leakage of fibrinogen and growth
factors into the interstitial space,
leukocyte aggregation and activation,
and obliteration of the cutaneous
lymphatic network can predispose a
patient to skin ulcers
(u) C. Increased capillary leakage
causes venous insufficiency.
(u) D. Inherited deficiency of protein C
predisposes patients to thrombosis.

109.

Diagnosis/Cardiology
A newborn is being evaluated for perioral cyanosis while feeding associated with
sweating. Vital signs are rectal temperature, 37.8 degrees C (100 degrees F), blood
pressure 80/45 mmHg, pulse 180/min, and respirations 40/min. A grade 3/6 harsh
systolic ejection murmur with a single loud S2 is heard at the left upper sternal border.
Electrocardiogram (ECG) shows right ventricular hypertrophy with right axis
deviation. Chest x-ray shows a boot- shaped heart and decreased pulmonary vascular
markings. Which of the following is the most likely diagnosis?
A. Atrial septal defect
B. Total anomalous pulmonary venous return
C. Coarctation of the aorta
D. Tetralogy of Fallot

Explanations
(u) A. Although the murmur may be
consistent with an ASD with pulmonary
hypertension the chest x-ray would
not show decreased pulmonary
vascular markings. With a large left to
right shunt large pulmonary arteries
and increased vascularity would be
seen.
(u) B. The murmur for TAPVR is a soft
systolic murmur at the left upper
sternal border with a split S2 in
addition to a short mid-diastolic
murmur at the low left sternal border.
(u) C. Cyanosis is usually not the
presenting sign for coarctation of the
aorta. Infants may present with heart
failure, ECG will show evidence of LVH.
(c) D. This is a common presentation for
tetralogy of fallot.

110.

Clinical Therapeutics/Cardiology
A hospitalized patient is found with confirmed pulseless ventricular tachycardia. IV access is
obtained following the second shock given. Which of the following medications is to be
administered immediately?
A. Amiodarone
B. Magnesium
C. Atropine
D. Epinephrine

Explanations
(u) A. Antiarrhythmics are
given after the third shock
and epinephrine has been
administered.
(u) B. Magnesium is useful
for torsades de pointes.
(u) C. Atropine may be used
for asystole or a slow
pulseless electrical activity
(PEA) rate.
(c) D. Epinephrine should be
given as soon as IV access is
obtained before or after the
second shock.

111.

Diagnosis/Cardiology
An electrocardiogram (ECG) shows a sinus rhythm with varying T wave heights, axis changes
every other beat and a wandering baseline. Which of the following is most likely the diagnosis?
A. Artifact
B. Digoxin toxicity
C. Pericardial effusion
D. Poor lead placement

Explanations
(u) A. Artifact could show a
wandering baseline, but not
the distinct axis changes.
(u) B. Digoxin toxicity can
cause bidirectional
tachycardia, but not
electrical alternans.
(c) C. This ECG pattern best
represents pericardial
effusion due to a swinging
heart in fluid and is known
as electrical alternans.
(u) D. Poor lead placement
would show different
patterns per the leads.

112.

Clinical Therapeutics/Cardiology
A 25 year-old female presents to the emergency department after an episode of substernal chest
pain with radiation to the middle of her back that came on suddenly and lasted for about four
minutes this morning while in bed. She denies previous episodes. Examination is unremarkable,
but she appears jittery. Toxicology screen is positive for cocaine. Which of the following
medications is contraindicated in this patient?
A. Lorazepam (Ativan)
B. Diltiazem (Cardizem)
C. Nitroglycerin (Nitrostat)
D. Propanolol (Inderal)

Explanations
(u) A. Lorazepam is not
contraindicated and can
help with agitation,
psychosis or seizures.
(u) B. Diltiazem is not
contraindicated but does
not have a definitive role in
treating cocaine toxicity.
(u) C. Nitroglycerin is not
contraindicated but does
not have a definitive role in
treating cocaine toxicity.
(c) D. Pure Beta blockers,
such as propranol, can
cause a paradoxical
hypertension because of
unopposed alphaadrenergic effects.

113.

Clinical Intervention/Cardiology
A 56 year-old female four days post myocardial infarction presents with a new murmur. On
examination the murmur is a grade 3/6 pansystolic murmur radiating to the axilla. She is
dyspenic at rest and has rales throughout all her lung fields. Blood pressure is 108/68 mmHg,
pulse 70 bpm. Which of the following would be the definitive clinical intervention?
A. Intra-aortic balloon counterpulsation
B. Mitral valve replacement
C. Coronary artery bypass surgery
D. Immediate fluid bolus

Explanations
(u) A. Although part of the
primary treatment to reduce
mitral regurgitation, it is not
definitive.
(c) B. MVR is the definitive
intervention to correct MR
caused by papillary muscle
rupture.
(u) C. CABG may be necessary
if significant blockage is found,
but it will not correct the mitral
regurgitation.
(u) D. A fluid bolus is indicated
if the patient is hypotensive.

114.

Clinical Therapeutics/Cardiology
A 16 year-old male with a history of tetralogy of Fallot presents to clinic for a follow-up visit
status post replacement of his right ventricle to pulmonary artery conduit. He has complaints
of chest pain with inspiration, fever and general malaise. Cardiac examination reveals a rub
with muffled heart sounds. Labs show an elevated erythrocyte sedimentation rate (ESR) and
leukocytosis. Which of the following is the most effective treatment?
A. Acetaminophen/oxycodone
B. Azithromycin
C. Indomethacin
D. Furosemide

Explanations
(u) A. See C for explanation.
(u) B. See C for explanation.
(c) C. Indomethacin is suitable
for controlling pain in
Dressler's syndrome. ASA is
preferred. Narcotics, diuretics
or antibiotics are not
recommended.
(u) D. See C for explanation.

115.

Diagnostic Studies/Cardiology
A 72 year-old male presents to the emergency department with crushing chest pain, dyspnea
and palpitations for 2 hours in duration. Enzymes are pending and he has been given aspirin
and sublingual nitroglycerin. He is rushed to the catheterization lab where they find a totally
occluded distal right coronary artery. Which of the following electrocardiogram (ECG) findings
supports the diagnosis?
A. Q waves in leads I, aVL, V5-V6
B. ST segment elevation in leads II, III, aVF
C. Hyperacute T waves in leads I, aVL
D. Flipped T waves with repolarization changes in leads V1-V4

Explanations
(u) A. Q waves in leads I, aVL,
V5-V6 represent infarction
involving the circumflex
artery.
(c) B. ST segment elevation in
leads II, III, aVF, represents an
acute process in the right
coronary artery.
(u) C. Hyperacute T waves in
leads I, aVL can represent the
initial changes of an infarction
involving the circumflex
artery.
(u) D. Flipped T waves with
repolarization changes in
leads V1-V4 can represent
early stages of infarction
involving the left anterior
descending artery.

116.

Health Maintenance/Cardiology
Which of the following population groups represent the greatest risk for
developing primary hypertension?
A. White non-Hispanic
B. Hispanic
C. Mediterranean
D. Black non-Hispanic

Explanations
(u) A. White non-Hispanic adults have a low risk of
hypertension compared to Hispanic and Black nonHispanics.
(u) B. Hispanic adults are lower risk of hypertension
than Black non-Hispanic, but not compared to White
non- Hispanic.
(u) C. Mediterranean adults have a lower risk of
hypertension than Black non-Hispanics.
(c) D. Black non-Hispanic adults have the highest risk
of hypertension.

117.

Diagnosis/Cardiology
A 60 year-old male with hypertension is brought to the emergency
department 30 minutes after the sudden onset of severe chest pain that
radiates to his back and arms. His blood pressure is 180/80 mmHg in his
left arm; no blood pressure reading can be obtained from the right arm.
ECG shows sinus tachycardia with left ventricular hypertrophy. A high
pitched decrescendo diastolic murmur is heard along the left mid-sternal
border. Which of the following is the most likely diagnosis?
A. Acute myocardial infarction
B. Aortic dissection
C. Pulmonary embolism
D. Right subclavian arterial embolus

Explanations
(u) A. Although included as part of the differential
the presentation is not consistent with AMI. ECG may
show ST changes and a murmur of mitral
regurgitation may be present with papillary muscle
rupture.
(c) B. This is a classic presentation for aortic
dissection.
(u) C. Patients will also present with shortness of
breath, feelings of impending doom and chest pain
that varies with respirations.
(u) D. Arterial embolus will present with symptoms
related to the location of the occlusion. Pain and
paresthesias are usually the earliest symptoms.

118.

History & Physical/Cardiology


Which of the following conditions is most suggestive of an abdominal
aortic aneurysm?
A. Abdominal mass
B. Hypertension
C. Chest pain
D. Syncope

Explanations
(c) A. An abdominal aortic aneurysm presents with a
pulsatile upper abdominal mass.
(u) B. Hypertension is not suggestive of an
abdominal aortic aneurysm.
(u) C. Abdominal aortic aneurysm presents with
midabdominal or lower back pain.
(u) D. Syncope is not common in abdominal aortic
aneurysm, unless it ruptures.

119.

Health Maintenance/Cardiology
Who is most likely to require subacute bacterial endocarditis (SBE)
prophylaxis prior to a dental procedure?
A. 22 year-old female with mitral valve prolapse
B. 36 year-old male with a bio-prosthesic mitral valve
C. 45 year-old female with an ASD closure 8 months ago with no residual
defect
D. 15 year-old male with a bicuspid aortic valve

Explanations
(u) A. See B for explanation.
(c) B. The AHA recommends that patients with
prosthetic heart valves receive antibiotic
prophylaxis. As should cardiac transplant recipients
with valve disease, unrepaired cyanotic CHD,
repaired CHD with prosthetic material or device
during the first six months of the procedure and
repaired CHD with residual defects at site of patch or
prosthetic device.
(u) C. See B for explanation.
(u) D. See B for explanation.

120.

Scientific Concepts/Cardiology
When evaluating jugular venous pulsations a prominent a wave represents
which of the following?
A. Atrial contraction against a closed tricuspid valve
B. Rapid filling of the right atrium
C. Tricuspid regurgitation
D. Poor left ventricle compliance

Explanations
(c) A. The a wave corresponds to right atrial
contraction.
(u) B. See A for explanation.
(u) C. See A for explanation.
(u) D. See A for explanation.

121.

History & Physical/Cardiology


An 18 year-old male high school basketball player comes to clinic for a routine
physical exam. His height is 193 cm (76 in.); arm span is 201 cm (79 in.). He has
long fingers and toes. Blood pressure is 146/62 mmHg and pulse is 64/min.
Which of the following exam findings is most consistent with the diagnosis?
A. Grade 2/6 high-frequency diastolic murmur at the third right intercostal
space
B. Grade 2/6 systolic ejection murmur at the second left intercostal space with
a fixed widely split S2
C. Grade 2/6 continuous murmur heard best at the high left sternal border
D. Grade 2/6 systolic murmur at the fourth left intercostal space that decreases
with squatting

Explanations
(c) A. This murmur is most consistent with aortic
regurgitation which can be present in patients
with Marfans syndrome and a dilated aortic
root.
(u) B. This murmur is most consistent with an
atrial septal defect.
(u) C. This murmur is most consistent with a
patent ductus arteriosus and unlikely in this
age patient.
(u) D. This murmur is most consistent with
hypertrophic obstructive cardiomyopathy.

122.

Scientific Concepts/Cardiology
Which of the following is the most likely initial effect on the left ventricle from
aortic stenosis?
A. Dilitation of the ventricle with diastolic dysfunction
B. Wall stiffness due to ischemia from decreased coronary blood flow
C. Paradoxical wall motion abnormalities due to increased preload
D. Concentric hypertrophy with preserved function

Explanations
(u) A. Dilitation of the ventricle is a later finding.
(u) B. This is not an effect from aortic stenosis
but coronary artery blockage.
(u) C. Preload is the end-diastolic pressure or
volume within the ventricle, ischemic heart
disease or infarct would cause paradoxical wall
motion abnormalities due to the increased
preload.
(c) D. Hypertrophy would be the initial changes
of the left ventricle as a response to the
increased pressure.

123.

Diagnosis/Cardiology
A 65 year-old female who recently had an anterior MI returns to clinic for
follow-up six weeks after. She has no chest pain, but reports decreased exercise
tolerance. Electrocardiogram (ECG) shows persistent ST elevation in leads V2V4. Which of the following is the most likely diagnosis?
A. Right ventricular infarction
B. Re-occlusion of the right coronary artery
C. Pericarditis
D. Ventricular aneurysm

Explanations
(u) A. RV infarction is present in one-third of
patients who had an inferior wall MI and
typically show ST elevation in V4 along with the
inferior leads.
(u) B. Occlusion of the right coronary artery
would show ST elevation in the inferior ECG
leads.
(u) C. Pericarditis shows ST elevation in multiple
leads with a history consistent of a viral illness
or fever.
(c) D. Persistent ST elevation in the leads where
a previous or recent anterior MI occurred is
most likely due to ventricular aneurysm.

124.

Diagnostic Studies/Cardiology
A 15 year-old male is brought to the emergency department 1 hour after an episode of
syncope while running in a 400-meter race. He had a similar episode 2 years ago. His
mother and maternal first cousin died suddenly at the ages of 32 and 17 years, respectively.
Examination shows abrasions of the face, hands, and knees. Neurologic examination
shows no abnormalities. Which of the following is the most appropriate initial test?
A. Tilt table test
B. CT scan of the head
C. Electrocardiogram
D. Electroencephalography

Explanations
(u) A. Tilt table testing should be
performed after structural heart
disease has been ruled out.
(u) B. CT scanning of the head
would not be the initial test of
choice.
(c) C. ECG is the most appropriate
to rule out any underlying rhythm
abnormalities, Holter or event
monitoring would also be included.
(u) D. EEG testing routinely would
not be helpful as an initial test in
this patient.

125.

Clinical Therapeutics/Cardiology
What is the optimal INR for a patient with a mechanical mitral valve prosthesis on
warfarin (Coumadin)?
A. 1.5-2.5
B. 2.0-3.0
C. 2.5-3.5
D. 3.5-4.5

Explanations
(u) A. See C for explanation.
(u) B. See C for explanation.
(c) C. Patients with mechanical MV
prostheses should maintain an INR
between 2.5-3.5. Aortic mechanical
valves can be maintained at an INR
of 2.0-3.0.
(u) D. See C for explanation.

126.

Clinical Intervention/Cardiology
Which of the following can optimize quality of life and is an definitive treatment for a
patient with refractory heart failure?
A. Ventricular assist device
B. Intra-aortic balloon counterpulsation
C. Cardiac transplantation
D. Partial resection of the left ventricle

Explanations
(u) A. Ventricular assist devices can
help to provide temporary
circulatory support to those
awaiting transplantation.
(u) B. IABC is useful in cardiogenic
shock, but does not improve
quality of life.
(c) C. Cardiac transplantation is
effective, with survival rates of 8090% in 1 year, 60-70% over 5 years.
It does improve quality of life
despite the immunosuppression
medications.
(u) D. Partial resection of the left
ventricle is still experimental and
has not shown to have successful
results.

127.

History & Physical/Cardiology


Which of the following physical exam findings suggests worsening or severe aortic
stenosis?
A. An ejection click preceding the murmur
B. A split S2 with respiration variation
C. Grade 2/6 systolic murmur radiating to the carotids
D. Palpable thrill over the right second intercostal space

Explanations
(u) A. This finding is typical in mild
to moderate AS.
(u) B. This can be a normal finding.
(u) C. This is the typical murmur for
aortic stenosis, but does not
suggest the degree of severity by
the grade.
(c) D. A palpable thrill or LV heave
with associated murmur suggests
severe AS.

128.

Scientific Concepts/Cardiology
Perfusion of the coronary arteries occurs primarily during which of the following?
A. Systole
B. Diastole
C. Afterload
D. Preload

Explanations
(u) A. See B for explanation.
(c) B. Coronary artery
perfusion occurs primarily
during diastole.
(u) C. See B for explanation.
(u) D. See B for
explanation.

129.

Clinical Therapeutics/Cardiology
What is the mechanism of action for aspirin?
A. Inhibits platelet aggregation by blocking cyclooxygenase-1 activity
B. Exerts antiplatelet effects by blocking the platelet membrane adenosine diphosphate receptors
C. Inhibits the platelet membrane glycoprotein IIb/IIIa receptor
D. Converts plasminogen to the natural fibrinolytic agent plasmin

Explanations
(c) A. Aspirin inhibits
platelet aggregation by
blocking cyclooxygenase-1
activity.
(u) B. This the mechanism
of action for ADP
antagonists such as
clopidogrel or ticlopidine.
(u) C. This is the mechanism
of action for glycoprotein
IIb/IIIa receptors such as
abciximab (ReoPro).
(u) D. This is the
mechanism of action for
tissue plasminogen
activators (tPA).

130.

History & Physical/Cardiology


A 56 year-old male with a 30 pack-year smoking history presents with substernal chest pain. The
pain is described as a pressure that radiates to his jaw. The pain has lasted consistently for 30
minutes with variable relief. His current medications include atorvastatin (Lipitor) and glyburide
(Micronase). Which of the following aggravating or relieving factors about the pain would support
the diagnosis?
A. Precipitated by exercise and relieved with rest
B. Unrelieved by nitroglycerin
C. Aggravated by deep breaths
D. Relieved with food

Explanations
(u) A. Precipitated by
exercise and relief with rest
is consistent with angina.
(c) B. Patient is having a
myocardial infarction which
is unrelieved by rest or
nitroglycerin. (u) C.
Aggravated by deep
breaths suggests
pericarditis.
(u) D. Relieved with food
suggests peptic ulcer
disease.

131.

Health Maintenance/Cardiology
A 26 year-old female presents to clinic with a left arm that is swollen and non-tender with bluish
discoloration along the upper arm and forearm. She is status post pacemaker insertion in the left
upper chest for third degree heart block, one week ago. Pulses are present and the arm is warm,
but not red. The pacemaker incision is healing well despite a hematoma and tenderness at the
site. Which of the following statements would be appropriate patient education about this
condition?
A. Reassurance that the discoloration is an expected finding.
B. Apply cold compresses to the site of the hematoma.
C. Elevation of the involved extremity will increase the swelling.
D. Aspirin should be taken to help manage pain.

Explanations
(c) A. This is indicative of
migratory ecchymosis and
expected after insertion of
a pacemaker.
(u) B. Warm compresses will
help to decrease the
hematoma.
(u) C. Elevation will help to
decrease the swelling.
(u) D. ASA is not the pain
medicine of choice in a
patient with a hematoma
due to its non-clotting
properties.

132.

Clinical Intervention/Cardiology
A 66 year-old female with a history of coronary artery disease presents with a new onset of
dizziness and fatigue for two weeks. She recalls nearly passing out on one occasion. Examination
is unremarkable except for bradycardia. Electrocardiogram (ECG) reveals a heart rate of 50 with a
normal PR interval followed by a normal QRS. There are several non-conducting P waves and no
lengthening of the PR interval. Which of the following interventions is the therapy of choice?
A. Permanent pacemaker
B. Radio-frequency ablation
C. Maze procedure
D. Automatic Implantable Cardioverter Defibrillator

Explanations
(c) A. This patient has
symptomatic second degree
type II heart block and
requires a pacemaker.
(u) B. Radio-frequency
ablation is useful for
supraventricular
tachyarrhythmias.
(u) C. Maze procedure is a
surgical procedure for
decreasing the incidence of
atrial fibrillation by creating
cuts simulating a pathway in
the atriums.
(u) D. Automatic
Implantable Cardioverter
Defibrillator (AICD) is used
in patients at risk for
sudden death.

133.

Diagnostic Studies/Cardiology
Pulmonary capillary wedge pressure indirectly measures which of the following?
A. Right ventricular end-diastolic pressure
B. Right atrial filling pressure
C. Left ventricular systolic pressure
D. Left atrial filling pressure

Explanations
(u) A. See D for explanation.
(u) B. See D for explanation.
(u) C. See D for explanation.
(c) D. Pulmonary capillary
wedge pressure indirectly
measures left atrial filling
pressures.

134.

History & Physical/Cardiology


A 26 year-old male presents with increased dyspnea with exercise. He has noted a decrease in his
exercise tolerance over the past several months. He denies chest pain or skipped heart beats.
Echocardiogram reveals left ventricular hypertrophy with asymmetric septal hypertrophy.
Ejection fraction is 65%. Which of the following is the most likely presenting history or physical
exam finding?
A. He has an older brother with the same diagnosis.
B. An S3 gallop is heard.
C. Patient notes completing a course of adriamycin.
D. Elevated jugular venous distension is noted.

Explanations
(c) A. Hypertrophic
cardiomyopathy can be
genetic and present in 25%
of first degree relatives.
(u) B. Dilated
cardiomyopathy may
present with an S3 gallop
due to volume overload.
(u) C. Adriamycin
chemotherapy can lead to
cardiac dysfunction, dilated
cardiomyopathy and
eventually heart failure.
(u) D. Restrictive or dilated
cardiomyopathy may
present with JVD due to
abnormalities of filling.

135.

Health Maintenance/Cardiology
Which of the following lifestyle recommendations would most benefit the heart
failure patient's quality of life?
A. Begin a regular exercise program
B. Total salt restriction
C. Home monitoring of blood pressure
D. Increase home oxygen requirements

Explanations
(c) A. Exercise training improves activity
tolerance and deconditioning offering
some recompensation.
(u) B. Although salt restriction is a
recommendation total salt restriction
would be unrealistic to achieve.
(u) C. Home monitoring of blood pressure
will not improve a heart failure patient's
quality of life.
(u) D. Increasing the requirements for
home oxygen could signal that they are
worsening and is palliative for their quality
of life.

136.

History & Physical/Cardiology


A newborn presents with blue discoloration of the peripheral extremities which
worsens with exposure to cold and improves with warming. Which of the following
is the most likely cause?
A. Raynauds phenomenon
B. Livedo reticularis
C. Erythromelagia
D. Acrocyanosis

Explanations
(u) A. Raynauds phenomenon has a
triphasic color response and is
exacerbated by cold or emotions.
(u) B. Livedo reticularis is characterized by
a lacy pattern on the skin of the lower
extremities.
(u) C. Erythromelagia is red, painful
extremities.
(c) D. Acrocyanosis is a blue discoloration of
the digits, usually of the peripheral
extremities in newborns, which worsens
with exposure to cold and improves with
warming.

137.

Diagnosis/Cardiology
A two month-old infant appeared well until three weeks ago when he became
dyspneic and had difficulty feeding. A 4/6 holosystolic murmur is heard at the left
lower sternal border in the 3rd ICS. An electrocardiogram (ECG) shows left and right
ventricular hypertrophy. Which of the following is the most likely diagnosis?
A. Atrial septal defect
B. Pulmonary hypertension
C. Ventricular septal defect
D. Tricuspid insufficiency

Explanations
(u) A. An ASD usually presents with a soft
mid-systolic murmur in the second left ICS
with a widely split and fixed S2.
(u) B. With pulmonary hypertension the
murmur may be most consistent with
pulmonary or tricuspid insufficiency.
(c) C. This is a classic presentation for a
ventricular septal defect.
(u) D. Tricuspid insufficiency is a systolic
murmur heard best at the 4th ICS that
may radiate to the apex.

138.

Diagnostic Studies/Cardiology
Which of the following laboratory tests would be elevated in a patient with
symptomatic heart failure?
A. Lactate dehydrogenase (LDH)
B. Troponin I (TnI)
C. C-reactive protein (CRP)
D. Brain natriuretic peptide (BNP)

Explanations
(u) A. LDH is not a sensitive marker in
patients with heart failure.
(u) B. Troponin measurements are specific
for myocardial infarctions.
(u) C. Increased serum levels of CRP are
found in patients with unstable angina
and MI. They can be a strong predictor of
coronary events.
(c) D. BNP is expressed in the ventricles
and is a sensitive assay in patients with
heart failure.

139.

Clinical Therapeutics/Cardiology
When prescribing loop diuretics which of the following electrolytes should be most closely
monitored?
A. Calcium
B. Potassium
C. Sodium
D. Chloride

Explanations
(u) A. See B for
explanation.
(c) B. Potassium along
with magnesium should
be monitored when
prescribing loop
diuretics.
(u) C. See B for
explanation.
(u) D. See B for
explanation.

140.

Diagnostic Studies/Cardiology
Which of the following studies is the best initial diagnostic evaluation to estimate ventricular size
and hypertrophy?
A. Electrocardiogram (ECG)
B. Cardiac CT scan
C. Echocardiogram
D. Myocardial perfusion imaging

Explanations
(u) A. ECG is not
sensitive or reliable to
estimate ventricular size
and hypertrophy.
(u) B. Cardiac CT scan
can detect coronary
calcification, but is most
sensitive to assess
disorders of the aorta.
(c) C. Echocardiogram
provides the safest and
most reliable means to
evaluate ventricular
size, hypertrophy and
function.
(u) D. Myocardial
perfusion imaging is
used for measurement
of LV ejection fraction
and assess regional wall
motion abnormalities.

141.

Diagnosis/Cardiology
A 44 year-old female presents to clinic for evaluation of a syncopal episode that occurred while
walking her dog two days ago. She denies amnesia or head trauma. She has had increasing dyspnea
on exertion and pedal edema. Physical examination reveals clubbing of her fingers and central
cyanosis. Auscultation of the heart reveals tricuspid insufficiency, widely split second heart sound
with a palpable P2. Echocardiogram reveals a large ostium secundum atrial septal defect with
bidirectional flow. Which of the following is a secondary complication in this patient?
A. Left heart failure
B. Ebsteins anomaly
C. Tricuspid stenosis
D. Pulmonary hypertension

Explanations
(u) A. This patient would
more likely have right
heart failure than left
heart failure.
(u) B. Ebsteins anomaly
is apical displacement of
the septal tricuspid
leaflet and not caused
by an ASD.
(u) C. Tricuspid stenosis
is not caused by an ASD.
(c) D. Her symptoms
and exam findings are
consistent with
pulmonary
hypertension and in
her case, Eisenmengers
disease, which is a late
finding.

142.

Clinical Intervention/Cardiology
Which of the following treatments will most benefit the diabetic
patient with two vessel coronary disease?
A. Stent placement
B. Percutaneous balloon angioplasty
C. Medical management
D. Coronary artery bypass graft

Explanations
(u) A. See D for explanation.
(u) B. See D for explanation.
(u) C. See D for explanation.
(c) D. CABG is the treatment of choice in a diabetic with
two or three vessel disease.

143.

Diagnosis/Cardiology
A 24 year-old male comes to the clinic with a one week history of pain
and swelling that involves the entire right upper extremity. He exercises
frequently and has noticed the pain worsening while lifting weights.
Examination shows enlarged cutaneous veins over the right anterior
chest wall with a palpable cord. His right hand appears dusky. Which of
the following is the most likely diagnosis?
A. Axillary-subclavian venous thrombosis
B. Thromboangiitis obliterans
C. Superficial thrombophlebitis of the cephalic vein
D. Brachial artery occlusion

Explanations
(c) A. Axillary-subclavian venous thrombosis can occur
in someone who strenuously exercises, has had a
central venous catheter or history of venous
thrombosis.
(u) B. Thromboangiitis obliterans involves the arteries,
although the smaller veins can be included and is
linked to tobacco use.
(u) C. This presentation is not consistent with
superficial thrombophlebitis and there is no history of
varicosities or previous IVs.
(u) D. Symptoms are not consistent with brachial
arterial occlusion.

144.

History & Physical/Cardiology


When performing a pre-participation sports physical in the adolescent
population, a murmur with which of the following qualities indicates a
risk for sudden death during exercise?
A. Increases with the Valsalva maneuver
B. Increases with squatting maneuver
C. Associated with a mid-systolic click
D. Mid-systolic without radiation to the carotids

Explanations
(c) A. Hypertrophic cardiomyopathy (HCM) is a known
cause of sudden death during or just after physical
exertion and competitive sports. The murmur
associated with HCM is worsened by conditions that
cause reduced ventricular volume such as the Valsalva
maneuver, sudden standing, and tachycardia.
(u) B. Typical systolic flow murmurs will become
accentuated with maneuvers which increase venous
blood flow to the heart and these murmurs do not
place athletes at risk for sudden cardiac death.
(u) C. Mitral valve prolapse is the most common type of
heart murmur that is associated with a mid-systolic
click. Mitral valve prolapse does not place the patient at
risk for sudden cardiac death.
(u) D. A mid-systolic heart murmur that fails to radiate
into the carotids is most commonly associated with a
benign systolic flow murmur and does not place the
athlete at risk for sudden cardiac death.

145.

Clinical Therapeutics/Cardiology
Which of the following antiarrhythmic drugs can be associated with
hyper- or hypothyroidism following long-term use?
A. Quinidine
B. Amiodarone
C. Digoxin
D. Verapamil

Explanations
(u) A. See B for explanation.
(c) B. Amiodarone is structurally related to thyroxine
and contains iodine, which can induce a hyper- or
hypothyroid state.
(u) C. See B for explanation.
(u) D. See B for explanation.

146.

Diagnosis/Cardiology
A 56 year-old male with a
known history of
polycythemia suddenly
complains of pain and
paresthesia in the left leg.
Physical examination reveals
the left leg to be cool to the
touch and the toes to be
cyanotic. The popliteal pulse
is absent by palpation and
Doppler. The femoral pulse is
absent by palpation but weak
with Doppler. The right leg
and upper extremities have
2+/4+ pulses throughout.
Given these findings what is
the most likely diagnosis?
A. Venous thrombosis
B. Arterialthrombosis
C. Thromboangiitis obliterans
D. Thrombophlebitis

Explanations
(u) A. See B for explanation.
(c) B.
Arterialthrombosishasoccurredandisevidencedbythelossofthepoplitealanddorsalispedispulse.Thisis
a surgical emergency. Venous occlusion and thrombophlebitis do not result in loss of arterial pulse.
(u) C. See B for explanation.
(u) D. See B for explanation.

147.

History & Physical/Cardiology


A 25 year-old female presents
with a three day history of
chest pain aggravated by
coughing and relieved by
sitting and leaning forward.
She is febrile and a CBC with
differential reveals
leukocytosis. Which of the
following physical
examination signs is
characteristic of her problem?
A. Pulsus paradoxus
B. Localized crackles
C. Pericardial friction rub
D. Wheezing

Explanations
(u) A. Pulsus paradoxus is a classic finding for cardiac tamponade.
(u) B. Localized crackles are associated with pneumonia and consolidation, not pericarditis.
(c) C. Pericardial friction rub is characteristic of an inflammatory pericarditis.
(u) D. Wheezing is characteristic for pulmonary disorders, such as asthma.

148.

Diagnosis/Cardiology
During physical examination
an elderly patient is noted to
have a painless, browncolored ulceration in the area
of the medial malleolus.
Which of the following is the
most likely diagnosis?
A. Arterial ulcer
B. Venous ulcer
C. Arterial insufficiency
D. Diabetic ulcer

Explanations
(u) A. Arterial ulcers typically are the last in the sequence of events of peripheral arterial disease;
which include decreased or absent pulses distal to the blockage, muscle atrophy, hair loss,
thickened nails, smooth and shiny skin, reduced skin temperature, pallor, cyanosis, ulcers, and
gangrene. Arterial ulcers typically occur on the feet in the areas of pressure points.
(c) B. Venous ulceration develops in the lower extremity secondary to venous incompetence and
chronic edema. The medial aspect of the ankle is the most common location.
(u) C. Arterial insufficiency is most likely to present with symptoms of claudication prior to the
development of skin ulcers. When these ulcers do occur, they are most commonly seen as arterial
ulcers and not venous ulcers.
(u) D. Diabetic ulcers typically occur due to atherosclerosis and arterial insufficiency along with
diabetic peripheral neuropathy. These ulcers are more likely to occur on pressure points on the foot
and fail to heal because of poor circulation to these areas.

149.

Health Maintenance/Cardiology
Which of the following is a proven risk factor for the
development of abdominal aortic aneurysm?
A. Infective endocarditis
B. Diabetes mellitus
C. Cigarette smoking
D. Alcohol abuse

Explanations
(u) A. Infective endocarditis is not associated with the development of
abdominal aortic aneurysm.
(u) B. Diabetic patients do have a higher rate of atherosclerosis, but there
is no clear causal evidence of diabetics being at higher risk for the
development of abdominal aortic aneurysm.
(c) C. Cigarette smoking is the primary risk factor for the development of
aortic aneurysms.
(u) D. Alcohol abuse is not related to development of abdominal aortic
aneurysm.

150.

Clinical Therapeutics/Cardiology
When instituting diuretic therapy for patients with
heart failure, which of the following is considered the
treatment of choice as first-line therapy in a failing
kidney due to its improved sodium clearance?
A. Hydrochlorothiazide (Diuril)
B. Bumetanide (Bumex)
C. Spironolactone (Aldactone)
D. Acetazolamide (Diamox)

Explanations
(u) A. Thiazide diuretics may have better hypertension control than the
short acting loop diuretics but they are generally ineffective when the
glomerular filtration rate falls below 30-40 mL/min.
(c) B. Loop diuretics remain active in severe renal insufficiency and are the
most effective type of diuretics used in the management of heart failure
symptoms. These agents have a rapid onset and result in natriuresis due
to their activity in the ascending limb in the Loop of Henle.
(u) C. Potassium-sparing diuretics have very weak diuretic properties and
are useful as adjunctive therapy in patients with Stage 3 or Stage 4 heart
failure who are already on a first-line diuretic and other agents.
(u) D. Acetazolamide is a carbonic anhydrase inhibitor and works by
causing a metabolic acidosis with loss of bicarbonate in the failing kidney.
It is a weak diuretic and does not work in the setting of a failing kidney.

151.

Diagnostic Studies/Cardiology
High ventricular filling pressures are indicated by a
rise in which of the following?
A. Brain natriuretic peptide
B. Troponin
C. Myoglobin
D. Creatinine

Explanations
(c) A. Brain natriuretic peptide (BNP) is a hormone released from the
myocardium when stretched such as with high ventricular filling
pressures from CHF. It can indicate or be used as prognostic evaluation in
patients with acute CHF.
(u) B. Troponin is a marker for cardiac muscle damage as in an acute
myocardial infarction or ischemia.
(u) C. Myoglobin is a byproduct of muscle destruction or damage from
ischemia or other causes of skeletal muscle damage. Myoglobin may rise
with high ventricular pressures, however it is nonspecific.
(u) D. Creatinine is a marker of renal function.

152.

Clinical Intervention/Cardiology
Which of the following is first-line treatment for a
symptomatic bradyarrhythmia due to sick sinus
syndrome?
A. Permanent pacemaker
B. Radiofrequencyablation
C. Antiarrhythmic therapy
D. Anticoagulation therapy

Explanations
(c) A. Permanent pacemakers are the therapy of choice in patients with
symptomatic bradyarrhythmias in sick sinus syndrome.
(u) B. Radiofrequency ablation is used for the treatment of accessory
pathways in the heart.
(u) C. See A for explanation.
(u) D. See A for explanation.

153.

Scientific Concepts/Cardiology
What is the most likely mechanism responsible for
retinal hemorrhages and neurologic complications in
a patient with infective endocarditis?
A. Metabolicacidosis
B. Embolizationofvegetations
C. Hypotension and tachycardia
D. Activation of the immune system

Explanations
(u) A. See B for explanation.
(c) B. The vegetations that occur during infective endocarditis can become
emboli and can be dispersed throughout the arterial system.
(u) C. See B for explanation.
(u) D. Glomerulonephritis and arthritis result from activation of the
immune system.

154.

Diagnostic Studies/Cardiology
Which of the following is a non-invasive
quick method of evaluating a patient with
suspected lower extremity arterial
insufficiency?
A. Ankle-Brachial Index
B. Striker Tonometry
C. CT Angiography
D. Lower extremity arteriography

Explanations
(c) A. The single most useful index is the ankle pressure. This can be obtained with an
Ankle-Brachial Index (ABI) which the severity of signs and symptoms of arterial
insufficiency are correlated to the findings on the ABI. It is a non-invasive study that
can be performed in an office setting.
(u) B. Striker Tonometry is used to evaluate compartment pressures.
(u) C. CT angiography is an invasive test involving radiation and contrast that cannot
be done in an office setting.
(u) D. Lower extremity arteriography is an invasive test involving radiation and
contrast that cannot be done in an office setting.

155.

History & Physical/Cardiology


Which of the following is the earliest
symptom for patients with left ventricular
failure?
A. Dependentedema
B. Dyspneaonexertion
C. Congestion
D. Chest pain

Explanations
(u) A. Right ventricular failure is manifested by dependent edema and congestion in
the lungs.
(c) B. Patients with left ventricular heart failure may be comfortable at rest and may
experience their first symptoms with dyspnea with conversation or with mild exertion.
(u) C. Right ventricular failure is manifested by congestion in the lungs with cough
being a possible manifestation of this congestion.
(u) D. Patients who have on-going left ventricular failure may have underlying
coronary heart disease. When the heart failure is progressing, chest pain (manifested
by angina) and myocardial infarction may be manifestations of ongoing, progressive
heart failure. Acute decline in heart failure may lead to acute myocardial infarction but
this tends to be a very late symptom.

156.

Clinical Therapeutics/Cardiology
When utilizing medical treatment for
hypertension, which of the following
classes of medications should be used with
caution in those with elevated potassium
levels or intrinsic renal disease?
A. Beta blockers
B. Calcium channel blockers
C. Central alpha agonists
D. ACE inhibitors

Explanations
(u) A. Beta blockers primarily work by decreasing cardiac contraction and slowing the
heart rate. There is no association with hyperkalemia.
(u) B. Calcium channel blockers (especially the dihydropyridines) act as vasodilators
with some effect at lessening cardiac contraction. Their use is not associated with
hyperkalemia.
(u) C. Central alpha agonists stimulate the alpha receptors in the brain resulting in
decreased vessel wall pressures. This process is responsible for causing a lowering of
the blood pressure. There is little effect on the kidney and these agents do not cause
hyperkalemia.
(c) D. ACE inhibitors are recognized as valuable agents in the management of
hypertension but they may result in hyperkalemia in patients with intrinsic renal
disease. They should be used with caution in patients who are taking potassiumsparing diuretics. These agents reduce glomerular filtration pressure resulting in
retention of potassium since less filtration occurs in the kidney.

157.

Clinical Intervention/Cardiology
A 60 year-old male has unstable angina, but
is otherwise healthy. A 90% lesion is found
in the left main coronary artery. Which of
the following interventions is most
appropriate?
A. Thrombolysis with t-PA
B. Medical management with nitrates
C. Coronary artery bypass graft (CABG)
D. Percutaneous transluminal coronary
angioplasty

Explanations
(u) A. Thrombolysis is recommended in acute embolic occlusion, not chronic.
(u) B. Medical management is appropriate only for patients who are not surgical
candidates.
(c) C. CABG is indicated in patients with stenosis of the left main coronary artery and
those with three-vessel coronary artery disease.
(u) D. Percutaneous transluminal coronary angioplasty is not the management of
choice in left mainstem artery disease because of increased potential complications
and mortality.

158.

Scientific Concepts/Cardiology
Which of the following pathogens has been linked with the
development of acute myocarditis?
A. Human papilloma virus
B. Rotavirus
C. Human Herpes Virus 6
D. Coxsackie B virus

Explanations
(u) A. Human papilloma virus most commonly is associated
with venereal warts and not myocarditis. (u) B. Rotavirus is
primarily responsible for acute diarrhea.
(u) C. Human Herpes Virus 6 is the causative organism for
Roseola.
(c) D. Although associated with a number of infectious and
systemic diseases, myocarditis is most frequently the result of
a viral infection, with Coxsackie B virus and echovirus being
the most frequently implicated in the infection.

159.

Diagnosis/Cardiology
A patient is having a routine physical examination. Funduscopic
examination reveals AV narrowing and venous nicking. The nasal
border of the optic disc appears blurred. Which of the following is
the most likely underlying cause?
A. Hypertension
B. Maculardegeneration
C. Retinal detachment
D. Diabetes mellitus

Explanations
(c) A. Hypertensive retinopathy may cause AV narrowing and
venous nicking due to these blood vessels having increased
pressures.
(u) B. Macular degeneration is associated with the formation
of Drusen and neoproliferation.
(u) C. Retinal detachment is observed funduscopically by the
retina being displaced from its attachment. Patients will
present with complaints of floaters in the eye field or abrupt
loss of vision if the detachment is complete.
(u) D. Diabetes mellitus is most closely associated with
neovascularization and microaneurysms as its primary
manifestations.

160.

History & Physical/Cardiology


A 33 year-old female presents to the office with a complaint of
palpitations. There is no history of any significant heart disease in
the past and her symptoms begin and end abruptly on their own.
Which of the following is the most likely explanation for these
symptoms?
A. Atrialfibrillation
B. Atrialflutter
C. Benign supraventricular tachycardia
D. Sinus tachycardia

Explanations
(u) A. Atrial fibrillation, the most common cause of sustained
irregular heart rates, tends to present in an older population
or in patients that have underlying heart disease such as
mitral stenosis.
(u) B. Atrial flutter is not commonly seen in young patients
without underlying heart disease although it does tend to be
intermittent in nature when it does occur.
(c) C. Benign supraventricular tachycardia tends to occur in a
young patient without pre-existing heart disease. Symptoms
begin and end abruptly without therapy and occurrences
happen only intermittently. It is the most likely diagnosis in
this setting.
(u) D. Sinus tachycardia may occur in a young patient
without underlying heart disease but the palpitations and
tachycardia tends to start and stop gradually rather than
abruptly.

161.

Diagnosis/Cardiology
A patient's EKG reveals widened P waves in lead II and large
negative deflection of the P wave in lead V1. Which of the
following is the most likely underlying cause for this?
A. Right atrial enlargement
B. Left atrial enlargement
C. Right ventricular hypertrophy
D. Left ventricular hypertrophy

Explanations
(u) A. Right atrial enlargement is found on EKG with peaked P
waves in lead II and a large positive deflection on the initial P
wave in lead V1.
(c) B. Wide P waves in lead II and a deep negative deflection
in lead V1 is due to P-mitrale which is caused by left atrial
enlargement.
(u) C. Right ventricular hypertrophy is noted on the EKG by
having a large R wave in lead AvR along with a deep S wave
in leads V5 or V6.
(u) D. On an EKG, left ventricular hypertrophy is associated
with tall R waves in leads V5 and V6, deep S waves in AvR and
V1, and tall R waves in AvL and AvF.

162.

Health Maintenance/Cardiology
According to the American College of Cardiology/American
Heart Association classification of heart failure, which of
the following patients fits the Stage B Classification system?
A. Asymptomatic patient with no structural disease or
patients who are at high risk for the development of heart
failure.
B. Asymptomatic patient with structural heart disease.
C. Symptomatic patient with structural heart disease.
D. Patients with refractory symptoms despite intervention.

Explanations
(u) A. According to the ACC/AHA 2005 guidelines, patients with risk
factors for heart disease but who have yet to develop symptoms are
categorized as Stage A. These patients have hypertension and lipid
disorders treated along with lifestyle modifications.
(c) B. According to the ACC/AHA 2005 guidelines, patients with
structural heart disease who have not yet experienced symptoms
are classified as Stage B. This is the initial stage in which medication
therapy other than just ACE inhibitors are recommended.
(u) C. According to the ACC/AHA 2005 guidelines, patients with
symptoms and structural heart disease are classified as Stage C.
(u) D. According to the ACC/AHA 2005 guidelines, patients with
refractory symptoms are classified as Stage D.

163.

Diagnosis/Cardiology
A 15 year-old male presents acutely to the office. His legs are
cool to the touch. Examination reveals that his pulses and
blood pressure are higher in the upper extremities than the
lower extremities. Femoral pulses are delayed and
weakened. Which of the following is the most likely
underlying diagnosis?
A. Pheochromocytoma
B. Conn's Syndrome
C. Cushing'sSyndrome
D. Coarctationoftheaorta

Explanations
(u) A. Pheochromocytoma is most commonly associated with
palpitations and feelings of warmth along with episodic (later
sustained) hypertension.
(u) B. Conn's Syndrome, also known as primary
hyperaldosteronism, is found in a patient with hypertension who
has unprovoked hypokalemia.
(u) C. Cushing's Syndrome is associated with hypertension but is
associated with the typical appearance of sustained elevated cortisol
levels such as purple striae, buffalo hump, and central obesity.
(c) D. Coarctation of the aorta typically has narrowing of the aorta
proximal to the left subclavian artery with resultant high blood
pressure in the upper extremities and decreased run off to the
lower extremities following this narrowed segment.

164.

Diagnostic Studies/Cardiology
Which of the following valvular heart abnormalities will
most likely be seen on echocardiography as a complication
of acute myocardial infarction?
A. Aortic stenosis
B. Aortic regurgitation
C. Mitral stenosis
D. Mitral regurgitation

Explanations
(u) A. Aortic stenosis puts additional strain on the left ventricle and
contributes to a patient developing an acute myocardial infarction
and does not occur as a result of one.
(u) B. Aortic regurgitation is not a consequence of acute myocardial
infarction and most commonly occurs as a result of an incompetent
valve or dilation of the proximal aorta.
(u) C. Mitral stenosis most commonly occurs as a complication of
rheumatic fever and not because of an acute myocardial infarction.
(c) D. In patients with acute myocardial infarction, echocardiogram
can show the severity of mitral regurgitation and the presence of
ventricular septal defect if one is present. Acute inferior wall
myocardial infarction is associated with acute mitral regurgitation
due to necrosis of the posterior papillary muscle which is supplied
by the right coronary artery.

165.

History & Physical/Cardiology


Which of the following
conditions is most closely
associated with an increased
intensity of the P2 heart sound?
A. Atrial septal defect
B. Aortic stenosis
C. Ventricular septal defect
D. Mitral valve prolapse

Explanations
(c) A. Atrial septal defect can cause a left to right shunt with resultant increased volume in the
right ventricle. The net result of this is that the P2 heart sound will be accentuated because of
the increased blood flow in the right ventricle and increased force of contraction in the right
ventricle to remove this blood. Atrial septal defect will also cause a fixed split S2 heart sound.
(u) B. Aortic stenosis primarily affects the left ventricle and causes left ventricular hypertrophy.
As the left ventricle contracts harder against increased valvular resistance, a systolic ejection
murmur occurs.
(u) C. Ventricular septal defect most commonly causes a holosystolic murmur as its most notable
ausculatory finding.
(u) D. Mitral valve prolapse most commonly is associated with a mid-systolic click with or without a
systolic heart murmur.

166.

Clinical Therapeutics/Cardiology
Which of the following
medication classes is considered
first-line therapy for patients
with Raynaud's phenomenon?
A. Beta blockers
B. Calcium channel blockers
C. Central alpha agonists
D. Oral nitrates

Explanations
(u) A. Beta blockers do not cause vasodilation against this vasospasm which makes them less
effective as treatment.
(c) B. Calcium channel blockers are first line therapy for patients who have uncontrolled
symptoms related to Raynaud's phenomenon. They are vasodilating agents which may play a
role in preventing the vasospasm that occurs with this disorder.
(u) C. See B for explanation.
(u) D. Second line agents for Raynaud's phenomenon may include topical but not oral nitrates.

167.

Scientific Concepts/Cardiology
Elevated levels of LP(a)
(Lipoprotein a) are considered to
be a risk factor for coronary
artery disease through which of
the following proposed
mechanisms?
A. Direct inhibition of HDL
B. Increasing the formation of
VLDL cholesterol
C. Competes for binding to the
plasminogen receptor
D. Enhancement of naturally
circulating triglycerides

Explanations
(u) A. See C for explanation.
(u) B. See C for explanation.
(c) C. Lp(a) lipoproteins are secreted by the liver, constitute 10% or less of the total plasma
lipoprotein mass, possess kringle domains homologous to plasminogen, and are associated with
vascular disease risk. Having domains homologous to plasminogen, Lp(a) will compete with
actual plasminogen for its receptor sites. Plasminogen activates plasmin, which facilitates
degradation of fibrin and matrix components. The main component of LP (a) is LDL, a known
risk factor for atherosclerosis.
(u) D. See C for explanation.

168.

Clinical Therapeutics/Cardiology
Which of the following
medication classes is the
recommended treatment for
patients who have an anterior
wall myocardial infarction with
poor left ventricular function?
A. Beta blockers
B. Calcium channel blockers
C. Potassium sparing diuretics
D. ACE inhibitors

Explanations
(u) A. Beta blockers need to be used with caution in a patient with severe left ventricular
dysfunction as they will worsen left ventricular contractility and may make this dysfunction
worse. They are used, however, in the early stages of chronic heart failure.
(u) B. Calcium channel blockers have no proven mortality benefit in patients with myocardial
infarctions and left ventricular dysfunction.
(u) C. Although potassium sparing diuretics are part of the later stage treatment of congestive
heart failure and tend to potentiate the other therapies, they are not first-line therapy in a
patient with left ventricular dysfunction.
(c) D. ACE inhibitors have been proven to be effective in the therapy of heart failure, especially
in the setting of left ventricular dysfunction. They are considered first-line therapy in patients
with symptomatic left ventricular systolic function.

169.

Diagnosis/Cardiology
The 35 year-old patient presents after a syncopal episode while
throwing a football with his son. Examination reveals regular
heart rate and EKG is normal. There were no symptoms prior
to the episode. Right radial pulse is decreased. Which of the
following is the most likely explanation for the syncope?
A. Carotid sinus hypersensitivity
B. Vasovagal episode
C. Cardiac dysrhythmia
D. Subclavian steal syndrome

Explanations
(u) A. Carotid sinus hypersensitivity typically presents with
syncope that is related to turning of the head (such as backing a
car out of a driveway) or from tight collars.
(u) B. Vasovagal episodes may result in syncope but there would
not be abnormal pulse findings.
(u) C. Cardiac dysrhythmia may occur in the setting of exercise
but this would not produce a decreased pulse on the affected
side.
(c) D. Subclavian steal syndrome occurs if the subclavian artery is
occluded proximal to the origin of the vertebral artery which
results in reversal in the direction of blood flow in the ipsilateral
vertebral artery. Exercise of the ipsilateral arm may increase
demand on the vertebral flow which produces a "subclavian
steal".

170.

Scientific Concepts/Cardiology
Which of the following is the most common complication that
occurs in the setting of acute pericarditis?
A. Pericardial effusion
B. Left ventricular failure
C. Superior vena cava syndrome
D. Subclavian steal syndrome

Explanations
(c) A. Accumulation of transudate, exudate or blood in the
pericardial sac can occur due to pericardial inflammation.
(u) B. Patients with acute pericarditis may have problems with
filling which affects the right ventricle more than the left ventricle.
(u) C. Patients with lung malignancy may develop superior vena
cava syndrome as a result of tumor invasion into the superior
vena cava.
(u) D. Patients with subclavian steal syndrome typically present
with arm ischemia and syncope and is not related to pericarditis.

171.

Diagnostic Studies/Cardiology
Which diagnostic study is considered to be the strategy of
choice for symptomatic patients with recurrent ischemia,
hemodynamic instability or impaired left ventricular
dysfunction?
A. Stress echocardiography
B. Exercise treadmill testing
C. Coronary angiography
D. Cardiac magnetic resonance imaging

Explanations
(h) A. Stress echocardiography should not be performed in the
setting of a patient who is acutely symptomatic.
(h) B. Exercise treadmill testing should not be performed in the
setting of an unstable patient with ongoing cardiac symptoms.
(c) C. Coronary or cardiac catheterization is the gold standard
technique in the evaluation of patients with significant cardiac
symptoms. Anatomical information along with degree of coronary
artery blockages are provided and patients may be able to
undergo coronary revascularization during or after this
procedure.
(u) D. Cardiac magnetic resonance imaging has limited availability
and is not part of national guidelines for evaluation of the cardiac
patient.

172.

History & Physical/Cardiology


Which of the following is a systemic manifestation of infective
endocarditis?
A. Hemarthrosis
B. Petechiae
C. Cafe au lait spots
D. Bronzing of the skin

Explanations
(u) A. Hemarthrosis is most commonly a consequence of a clotting
disorder such as hemophilia.
(c) B. Petechiae, splinter hemorrhages, Janeway lesions, and
Osler's nodes are systemic manifestations of patients who have
infective endocarditis.
(u) C. Cafe au lait spots are seen in Neurofibromatosis (von
Recklinghausen's syndrome).
(u) D. Bronzing of the skin is most commonly associated with
hemochromatosis or Addison's disease.

173.

Diagnostic Studies/Cardiology
A 76 year-old male presents after returning from a Safari in
Africa. Seven days ago he experienced chest pressure lasting
one hour that did not respond to three sublingual
nitroglycerin tablets. There was no ability to have lab work or
an EKG. The pain has not returned. If the patient had a nonSTEMI myocardial infarction, which of the following studies
will still be positive?
A. Electrocardiogram
B. Myoglobulin
C. CK-MB index
D. Troponin I

Explanations
(u) A. Patients suffering from a non-STEMI myocardial infarction
will not develop Q waves and most likely will have a normal EKG
five days after an acute event.
(u) B. Myoglobulin is a nonspecific enzyme that is released into
the circulation after any skeletal muscle damage, including a
myocardial infarction. It is the first enzyme that becomes positive
in the setting of acute myocardial infarction but its non-specific
measurement makes it less useful in the setting of acute
myocardial infarction. It returns to baseline within 24 hours after
infarction.
(u) C. CK-MB index has improved sensitivity for myocardial
muscle damage that occurs with acute myocardial infarction but
it returns to baseline within 2-3 days after injury.
(c) D. Troponin I levels will stay positive for at least one week
following myocardial infarction and is the preferred enzyme to
measure in this setting.

174.

Clinical Intervention/Cardiology
A 52 year-old patient with episodes of syncope has an
electrocardiogram which shows a consistently prolonged PR
interval with a missing QRS every two beats. Which of the
following is the most effective management?
A. Permanent pacing
B. Beta-blocker
C. ACEInhibitor
D. Defibrillation

Explanations
(c) A. This is consistent with ECG findings of a Mobitz type II AV
block. Since the patient is symptomatic this type of AV block
requires a permanent pacing to prevent total AV disassociation.
(u) B. Beta-blockers will slow conduction from the AV node and is
not indicated with this type of AV block. (u) C. There is no
indication for ACE Inhibitors in Mobitz Type II heart block.
(u) D. Defibrillation is not indicated in a person with AV block.

175.

Health Maintenance/Cardiology
According to the Joint National Commission VII Guidelines,
blood pressure targets are lower in patients with diabetes
mellitus and what other condition?
A. Liver disease
B. Renal disease
C. Thyroid disease
D. Peripheral vascular disease

Explanations
(u) A. See B for explanation.
(c) B. Blood pressure targets for hypertensive patients at the
greatest risk for cardiovascular events, particularly those with
diabetes and chronic kidney disease, are lower (less than
130/80) than for those individuals with lower cardiovascular risk
(goal is less than 140/90).
(u) C. See B for explanation.
(u) D. See B for explanation.

176.

Diagnosis/Cardiology
A patient presents to the office following a syncopal episode.
The patient claims that the syncope occurs when he changes
position such as rolling over in bed or when he bends over to
tie his shoes. Which of the following is the most likely
explanation for this presentation?
A. Carotid sinus hypersensitivity
B. Vasovagal episode
C. Subclavian steal syndrome
D. Atrial myxoma

Explanations
(u) A. Carotid sinus hypersensitivity may present with syncope
but is usually related to tight collars or when excessively turning
the head.
(u) B. Vasovagal episodes may occur with syncope as its
manifestation but it is not caused by changes in position.
(u) C. Subclavian steal syndrome may present with syncope that
is related to exercise of the affected arm which results in a
decreased pulse when the Adson maneuver is performed.
(c) D. Atrial myxoma most commonly presents with sudden
onset of symptoms that are typically positional in nature due to
the effect that gravity has on the tumor. Myxomas are the most
common type of primary cardiac tumor in all age groups and are
most commonly found in the atria.

177.

Clinical Therapeutics/Cardiology
Which of the following is the optimal therapy for a 76 year-old
patient with no allergies who has chronic atrial fibrillation?
A. Aspirin
B. Clopidogrel (Plavix)
C. Warfarin (Coumadin)
D. Low molecular weight heparin

Explanations
(u) A. Aspirin's role to prevent thromboembolism in atrial
fibrillation is limited to patients with no risk factors who are under
age 65.
(u) B. Clopidogrel is not the optimal therapy for patients with
atrial fibrillation.
(c) C. Patients older than age 75 who have chronic atrial
fibrillation should be anticoagulated with warfarin to maintain an
INR between 2.5 and 3.0 for optimum therapy unless a
contraindication to therapy exists.
(u) D. Due to the increased costs and need for parenteral
therapy, daily subcutaneous heparin is not first line therapy
unless warfarin therapy is contraindicated.

178.

Clinical Intervention/Cardiology
Patients who undergo percutaneous angioplasty or who have
coronary artery revascularization often are treated with
glycoprotein IIb/IIIa inhibitors. What is the major side effect
associated with these agents?
A. Hypotension
B. Bleeding
C. Coronary vasospasm
D. Acute renal failure

Explanations
(u) A. See B for explanation.
(c) B. Glycoprotein IIb/IIa inhibitors have their activity in the final
stages of platelet bridging and are associated with bleeding when
used in the management of acute myocardial infarction. Since
they are effective at treating and preventing new clot formation,
bleeding is the main concern and complication with the use of
these agents.
(u) C. See B for explanation.
(u) D. See B for explanation.

179.

Health Maintenance/Cardiology
Which of the following is an absolute contraindication for the
performance of exercise stress testing for patients who wish to
start an exercise program?
A. Second degree heart block type 1
B. Severe aortic stenosis
C. Atrial fibrillation with controlled ventricular response
D. Recent diagnosis of lung cancer

Explanations
(u) A. See B for explanation.
(c) B. Contraindications to stress testing include rest angina within
the last 48 hours, unstable cardiac rhythm, hemodynamically
unstable patient, severe aortic stenosis, acute myocarditis,
uncontrolled heart failure, and active infective endocarditis.
(u) C. See B for explanation.
(u) D. See B for explanation.

180.

History & Physical/Cardiology


A 23 year-old male with recent upper respiratory symptoms
presents complaining of chest pain. His pain is worse lying
down and better sitting up and leaning forward.
Electrocardiogram shows widespread ST segment elevation.
Which of the following is the most likely physical examination
finding in this patient?
A. Elevated blood pressure
B. Subungual hematoma
C. Diastolic murmur
D. Pericardial friction rub

Explanations
(u) A. Acute pericarditis is usually not associated with elevated
blood pressure. One would expect to see hypertensive pressures
in the setting of an aortic dissection.
(u) B. Subungual hematomas are usually seen in endocarditis not
pericarditis.
(u) C. A diastolic murmur in a patient with chest pain would likely
be associated with acute aortic regurgitation in the setting of an
aortic dissection.
(c) D. This patient has symptoms consistent with acute pericarditis
and would most likely have a pericardial friction rub on
examination.

181.

Diagnostic Studies/Cardiology
A 53 year-old male with history of hypertension presents complaining of recent 4/10
left-sided chest pain with exertion that is relieved with rest. He states the pain usually
lasts approximately 4 minutes and is relieved with rest. Heart examination reveals
regular rate and rhythm with no S3, S4, or murmur. Lungs are clear to auscultation
bilaterally. Electrocardiogram reveals no acute changes. Which of the following is the
most appropriate initial step in the evaluation of this patient?
A. Cardiac catheterization
B. CT Angiogram of the chest
C. Echocardiogram
D. Nuclear stress test

Explanations
(u) A. This patient has signs and
symptoms consistent with stable
angina. Noninvasive diagnostic testing
is preferred in this patient.
(u) B. CT angiogram may be useful for
the evaluation of chest pain, however
its role in routine practice has not been
established.
(u) C. This patient has signs and
symptoms of stable angina. There are
no signs of valvular heart disease on
examination. While an echocardiogram
may be performed at some point, it is
not the best initial diagnostic step to
determine the etiology of the patient's
angina.
(c) D. Nuclear stress testing is the most
appropriate initial diagnostic study in
the evaluation of a patient with signs
and symptoms consistent with stable
angina.

182.

Clinical Therapeutics/Cardiology
A 48 year-old male with diabetes mellitus presents for routine physical examination. Of
note his blood pressure each of his last two follow-up visits was 150/90 mmHg. Today
the patient's BP is 148/88 mmHg. The patient denies complaints of chest pain, change in
vision, or headache. Which of the following is the most appropriate management for
this patient?
A. Atenolol (Tenormin)
B. Nifedipine (Procardia)
C. Hydralazine (Apresoline)
D. Lisinopril (Zestril)

Explanations
(u) A. See D for explanation.
(u) B. See D for explanation.
(u) C. See D for explanation.
(c) D. ACE inhibitors are the first line
treatment of choice in a patient with
hypertension and diabetes.

183.

Health Maintenance/Cardiology
A 36 year-old female presents for a routine physical. She has no current complaints and
her only medication is oral contraceptives. The patient is preparing for a trip to
Australia and is worried about the long flight as her mom has a history of deep vein
thrombosis after a long trip several years ago. Physical examination reveals BP 110/60
mmHg, HR 66 bpm, regular. Heart is regular rate and rhythm without murmur, lungs
are clear to auscultation bilaterally and extremities are without edema. Which of the
following is the most appropriate recommendation for your patient?
A. Discontinue oral contraceptives
B. Recommend walking frequently during the flight
C. Begin daily aspirin therapy
D. Increase fluid intake 2-3 days prior to the flight

Explanations
(u) A. See B for explanation.
(c) B. The risk of deep vein thrombosis
after air travel increases with flight
duration. Preventive measures for
patients include using support hose
and performing in-flight exercises and
walking.
(u) C. See B for explanation.
(u) D. See B for explanation.

184.

Diagnostic Studies/Cardiology
A 3 month-old female presents with her mom for physical examination. The
patient's mom denies any complaints. On examination you note a welldeveloped, well-nourished infant in no apparent distress. There is no
cyanosis noted. Heart examination reveals a normal S1 with a
physiologically split S2. There is a grade III/VI high-pitched, harsh,
pansystolic murmur heard best at the 3rd and 4th left intercostal spaces
with radiation across the precordium. Which of the following is the initial
diagnostic study of choice in this patient?
A. CT angiogram
B. Electrocardiogram
C. Echocardiogram
D. Cardiac catheterization

Explanations
(u) A. This patient has signs and symptoms
consistent with a ventricular septal defect (VSD).
CT angiogram and electrocardiogram are not
indicated in establishing the diagnosis of a VSD.
(u) B. See A for explanation.
(c) C. Echocardiogram is the initial diagnostic study
of choice in the diagnosis of a VSD.
(u) D. Cardiac catheterization may be necessary to
accurately measure pulmonary pressures or if a
VSD can not be well localized on echocardiogram,
but it is not the initial diagnostic study of choice.

185.

Clinical Intervention/Cardiology
A 20 year-old male presents with complaint of brief episodes of rapid heart
beat with a sudden onset and offset that have increased in frequency. He
admits to associated shortness of breath and lightheadedness. He denies
syncope. Electrocardiogram reveals a delta wave prominent in lead II. Which
of the following is the most appropriate long-term management in this
patient?
A. Implantable cardio defibrillator
B. Radiofrequency ablation
C. Verapamil (Calan)
D. Metoprolol (Lopressor)

Explanations
(u) A. Implantable cardio defibrillators are
indicated in the treatment of ventricular
arrhythmias, not Wolf-Parkinson- White (WPW)
syndrome.
(c) B. Radiofrequency ablation is the procedure of
choice for long-term management in patients with
accessory pathways (WPW) and recurrent
symptoms.
(u) C. Calcium channel blockers and beta-blockers
are not the best options for the long-term
management of WPW. They may decrease the
refractoriness of the accessory pathway or
increase the refractoriness of the AV node in
patients with atrial fibrillation or atrial flutter who
have an antegrade conducting bypass tract. This
may lead to faster ventricular rates.
(u) D. See C for explanation.

186.

Diagnosis/Cardiology
A 60 year-old female recently discharged after an 8 day hospital stay for
pneumonia presents complaining of pain and redness in her right arm. The
patient thinks this was the area where her IV was placed. The patient denies
fever or chills. Examination of the area reveals localized induration,
erythema and tenderness. There is no edema or streaking noted. Which of
the following is the most likely diagnosis?
A. Acute thromboembolism
B. Thrombophlebitis
C. Cellulitis
D. Lymphangitis

Explanations
(u) A. Acute thromboembolism is usually
associated with edema of the extremity and warm
temperature. Thromboembolism is unusual after
thrombophlebitis.
(c) B. This patient's signs and symptoms are
consistent with thrombophlebitis. Short-term
venous catheterization of a superficial arm vein is
commonly the cause and thrombophlebitis
characterized by dull pain, induration, redness
and tenderness along the course of the vein.
(u) C. Cellulitis is usually associated with fever,
increased warmth over the affected area and
associated edema. (u) D. Lymphangitis is
associated with fever, malaise, chills, and streaking.

187.

History & Physical/Cardiology


A patient with a history of chronic venous insufficiency presents for routine
follow-up. Which of the following findings is most likely on physical
examination?
A. Cold lower extremities
B. Diminished pulses
C. Lower extremity edema
D. Palpable cord

Explanations
(u) A. Cold lower extremities are more commonly
seen in peripheral arterial, not venous, disease.
(u) B. Diminished pulses are seen in peripheral
arterial disease.
(c) C. Patients with chronic venous insufficiency will
commonly have lower extremity edema.
(u) D. A palpable cord is more common in
superficial thrombophlebitis.

188.

Diagnostic Studies/Cardiology
A 29 year-old female with history of IV drug abuse presents with ongoing fevers for three weeks. She
complains of fatigue, worsening dyspnea on exertion and arthralgias. Physical examination reveals a
BP of 130/60 mmHg, HR 90 bpm, regular, RR 18, unlabored. Petechiae are noted beneath her
fingernails. Fundoscopic examination reveals exudative lesions in the retina. Heart examination
shows regular rate and rhythm, there is a grade II-III/VI systolic murmur noted, with no S3 or S4.
Lungs are clear to auscultation bilaterally, and the extremities are without edema. Which of the
following is the diagnostic study of choice in this patient?
A. Electrocardiogram
B. CT angiogram of the chest
C. Cardiac catheterization
D. Transesophageal echocardiogram

Explanations
(u) A. See D for
explanation.
(u) B. See D for
explanation.
(u) C. See D for
explanation.
(c) D. This patient's
signs and symptoms
are consistent with
infective endocarditis.
The diagnostic study of
choice would be a
transesophageal
echocardiogram.

189.

Clinical Therapeutics/Cardiology
A 49 year-old female presents complaining of several episodes of chest pain recently. Initial ECG in the
emergency department shows no acute changes. Two hours later, while the patient was having pain,
repeat electrocardiogram revealed ST segment elevation in leads II, III, and AVF. Cardiac
catheterization shows no significant obstruction of the coronary arteries. Which of the following is
the treatment of choice in this patient?
A. Nifedipine (Procardia)
B. Metoprolol (Lopressor)
C. Lisinopril (Zestril)
D. Carvedilol (Coreg)

Explanations
(c) A. This patient is
most likely having
coronary artery spasm.
This can be treated
prophylactically with
calcium channel
blockers such as
nifedipine.
(h) B. Beta-blockers
may exacerbate the
symptoms of coronary
vasospasm.
(u) C. ACE inhibitors
are not effective in the
treatment or
prevention of coronary
vasospasm.
(u) D. Carvedilol is not
effective in the
treatment or
prevention of coronary
vasospasm.

190.

History & Physical/Cardiology


A 75 year-old female with a history of long-standing hypertension presents
with shortness of breath. On examination you note a diastolic murmur at
the left upper sternal border. Which of the following maneuvers would
accentuate this murmur?
A. Sitting up and leaning forward
B. Lying on left side
C. Performing Valsalva maneuver
D. Standing upright

Explanations
(c) A. This patient has history findings consistent
with aortic insufficiency which is characterized by a
diastolic murmur that is accentuated when the
patient sits up and leans forward.
(u) B. The left lateral decubitus position
accentuates the murmur of mitral stenosis.
(u) C. Valsalva and standing maneuvers help to
differentiate the murmurs associated with aortic
stenosis and hypertrophic cardiomyopathy.
(u) D. See C for explanation.

191.

Diagnosis/Cardiology
A 50 year-old male with history of alcohol abuse presents with complaint of
worsening dyspnea. Physical examination reveals bibasilar rales, elevated
jugular venous pressure, an S3 and lower extremity edema. Chest x-ray
reveals pulmonary congestion and cardiomegaly. Electrocardiogram shows
frequent ventricular ectopy. Echocardiogram shows left ventricular
dilatation and an ejection fraction of 30%. Which of the following is the
most likely diagnosis in this patient?
A. Hypertrophic cardiomyopathy
B. Dilated cardiomyopathy
C. Restrictive cardiomyopathy
D. Tako-Tsubo cardiomyopathy

Explanations
(u) A. Hypertrophic cardiomyopathy is
characterized by a hyperdynamic left ventricle
with asymmetric left ventricular hypertrophy.
(c) B. Dilated cardiomyopathy is often caused by
chronic alcohol use. It is characterized by signs and
symptoms of left-sided heart failure, a dilated left
ventricle and decreased ejection fraction.
(u) C. Restrictive cardiomyopathy is characterized
more commonly by right-sided heart failure than
by left-sided heart failure. There is rapid early
filling with diastolic dysfunction. Patients with
restrictive cardiomyopathy will have a small
thickened left ventricle and a normal or near
normal ejection fraction on echocardiogram.
(u) D. Tako-Tsubo cardiomyopathy (broken heart
syndrome) is characterized by signs and
symptoms of acute coronary syndrome, ST
segment elevation on ECG and left ventricular
apical dyskinesia.

192.

Clinical Therapeutics/Cardiology
A 76 year-old active female with history of hypertension and
hypothyroidism presents with complaints of palpitations and dyspnea on
exertion. On examination vital signs are BP 120/80 mmHg, HR 76 bpm,
irregular, RR 16. Heart examination reveals an irregularly, irregular rhythm
without murmur. Lungs are clear to auscultation and extremities are
without edema. Which of the following is the most important medication
to initiate for chronic therapy in this patient?
A. Warfarin (Coumadin)
B. Verapamil (Calan)
C. Amiodarone (Cordarone)
D. Digoxin (Lanoxin)

Explanations
(c) A. Anticoagulation is necessary in all patients
with atrial fibrillation to prevent thromboembolic
events unless there is contraindication.
(u) B. This patient currently has a controlled
ventricular rates and does not require chronic
calcium channel blockers or digoxin at this time.
(u) C. Antiarrhythmic therapy may be indicated in
some patients with atrial fibrillation, but
anticoagulation is indicated in all patients unless
there is contraindication.
(u) D. See B for explanation.

193.

Diagnosis/Cardiology
A 58-year old male presents for a six week follow-up after an acute
anterior wall myocardial infarction. He denies chest pain and
shortness of breath. Electrocardiogram shows persistent ST segment
elevation in the anterior leads. Echocardiogram reveals a sharply
delineated area of scar that bulges paradoxically during systole.
Which of the following is the most likely diagnosis in this patient?
A. Left ventricular aneurysm
B. Postinfarction ischemia
C. Ischemic cardiomyopathy
D. Constrictive pericarditis

Explanations
(c) A. Left ventricular (LV) aneurysm develops in about 1020 percent of patients following acute myocardial
infarctions, especially anterior wall myocardial infarctions.
LV aneurysm is identified by ST segment elevation that is
present beyond 4-8 weeks after the acute infarct and a
scar that bulges paradoxically during systole on
echocardiogram.
(u) B. Postinfarction ischemia is recurrent ischemia that is
more common in patients with non-ST segment elevation
myocardial infarctions and is characterized by
postinfarction angina. This patient denies any chest pain.
(u) C. Ischemic cardiomyopathy would be characterized by
decreased ejection fraction on echocardiogram and wall
motion abnormalities. Ischemic cardiomyopathy is not
associated with ST segment elevation or bulge of scar on
echocardiogram.
(u) D. Constrictive pericarditis is characterized by signs
and symptoms of right-sided heart failure with increased
jugular venous pressures and a septal bounce on
echocardiogram.

194.

History & Physical/Cardiology


A 75 year-old female with history of coronary artery disease and
dyslipidemia presents for routine follow-up. Physical examination
reveals loss of hair on the lower extremities bilaterally with thinning
of the skin. Femoral pulses are +2/4 bilaterally, pedal pulses are
diminished bilaterally. Ankle brachial index is reduced. Which of the
following signs or symptoms is this patient most likely to have?
A. Lower extremity edema
B. Calf pain with walking
C. Numbness of the lower extremities
D. Itching of the lower extremities

Explanations
(u) A. This patient has signs and symptoms consistent with
arterial insufficiency. Lower extremity edema is seen in
patients with venous insufficiency.
(c) B. This patient has signs and symptoms consistent with
arterial insufficiency and would most likely complain of
intermittent claudication.
(u) C. Numbness of the lower extremities would be seen
with acute arterial occlusion.
(u) D. Itching of the lower extremities may be seen in
chronic venous insufficiency because of secondary skin
changes, but is not common in arterial insufficiency.

195.

Diagnosis/Cardiology
A 55 year-old male with history of hypertension and diabetes
mellitus presents to the emergency department. The patient's wife
states that the patient developed progressive irritability and
confusion today after complaining of a headache. Physical
examination reveals a BP of 230/130 mmHg and papilledema. Which
of the following is the most accurate diagnosis in this patient?
A. Resistant hypertension
B. Hypertensive urgency
C. Hypertensive emergency
D. Malignant hypertension

Explanations
(u) A. Resistant hypertension is the failure to reach blood
pressure control in patients who are compliant with a 3
drug regimen including a diuretic.
(u) B. Hypertensive urgency is a systolic BP > 220 or a
diastolic BP > 125 in a patient who is asymptomatic or who
has disk edema, progressive target organ complications.
Hypertensive urgency must be treated within a few hours
of presentation.
(u) C. Hypertensive emergency is similar to hypertensive
urgency, however the BP is significantly elevated and must
be lowered within an hour.
(c) D. Malignant hypertension is significantly elevated BP
with progressive retinopathy, including papilledema,
encephalopathy, and headache.

196.

Scientific Concepts/Cardiology
A patient undergoes biopsy for suspected myocarditis. Which of the
following is the most likely etiologic agent?
A. West Nile virus
B. Rhinovirus
C. Coxsackie B virus
D. Cytomegalovirus

Explanations
(u) A. See C for explanation.
(u) B. See C for explanation.
(c) C. Coxsackie B virus, Hepatitis C, adenovirus, and HIV
are the predominant agents in clinically significant acute
viral myocarditis in the US.
(u) D. See C for explanation.

197.

Health Maintenance/Cardiology
A 65 year-old male with coronary artery disease, hypertension, and diabetes mellitus is
admitted with dyspnea and lower extremity edema. The chest x-ray reveals small bilateral
pleural effusions. Echocardiogram shows an ejection fraction of 30% with no valvular heart
disease. The patient is treated in the hospital with furosemide (Lasix) and lisinopril (Zestril).
What education should be given to this patient upon discharge to help prevent
readmission?
A. Elevate the head of bed at home
B. Avoid physical activity
C. Monitor daily weights
D. Restrict fluid intake

Explanations
(u) A. Elevation of the head of the
bed may help the patient if they
have symptoms of dyspnea but it
will not help prevent readmission
to the hospital with a CHF
exacerbation.
(u) B. In stable patients increasing
physical activity or regular
exercise can diminish symptoms.
(c) C. Strategies to prevent
rehospitalization can include
monitoring daily weights, case
management and patient
education regarding selfadjustment of diuretics.
(u) D. Fluid restriction is not
helpful in the readmission for
CHF.

198.

History & Physical/Cardiology


A 60 year-old male nonsmoker with history of coronary artery disease presents with
complaint of worsening dyspnea on exertion for three weeks. He admits to orthopnea and
lower extremity edema, but denies chest pain, palpitations, and syncope. The patient's last
echocardiogram revealed an ejection fraction of 30%. Which of the following would you
most likely find on physical examination?
A. Pericardial friction rub
B. Third heart sound
C. Accentuated first heart sound
D. Mid-systolic click

Explanations
(u) A. A pericardial friction rub is a
sign of pericarditis, not heart
failure.
(c) B. An S3 on physical
examination is consistent with
heart failure.
(u) C. An accentuated first heart
sound is noted in tachycardia,
short PR interval rhythms,
increased cardiac output states
and mitral stenosis.
(u) D. A mid-systolic click is noted
in patients with mitral valve
prolapse.

199.

Clinical Therapeutics/Cardiology
A 78 year-old male with history of coronary artery disease s/p coronary artery bypass
grafting, hypertension, and dyslipidemia presents for routine physical examination. He feels
well except for occasional brief episodes of substernal chest pain with exertion that are
relieved with rest. He denies associated dyspnea, nausea or diaphoresis. Physical
examination reveals a BP of 110/70 mmHg, HR 56 bpm, regular, RR 14, unlabored. Lungs are
clear to auscultation, heart is bradycardic, but regular with no S3, S4 or murmur.
Electrocardiogram done in the office shows no acute ST-T wave changes. Which therapy is
indicated for the acute management of this patient's symptoms?
A. Sublingual nitroglycerine
B. Metoprolol (Lopressor)
C. Verapamil (Calan)
D. Lisinopril (Zestril)

Explanations
(c) A. Sublingual nitroglycerine is
the drug of choice for the acute
management of chronic stable
angina.
(u) B. Beta-blockers are
preventative and not the first
choice for the acute management
of chronic stable angina. Betablockers may worsen this
patient's bradycardia.
(u) C. Calcium channel blockers
are the third-line antiischemic
agent and may also reduce the
patient's heart rate.
(u) D. ACE inhibitors will not
provide acute relief of anginal
symptoms.

200.

Scientific Concepts/Cardiology
A 36 year-old male presents for follow-up of his hypertension. The patient is currently on
three anti-hypertensive medications without improvement of his blood pressure. On
examination his BP is 170/86mmHg and his HR is 60bpm and regular. His heart examination
reveals a regular rate and rhythm without S3, S4 or murmur and his lungs are clear to
auscultation bilaterally. Abdominal examination reveals a bruit over his left upper
abdomen. Which of the following is the most likely underlying etiology for this patient's
hypertension?
A. Pheochromocytoma
B. Renal artery stenosis
C. Cushing syndrome
D. Coarctation of the aorta

Explanations
(u) A. Pheochromocytoma is an
uncommon cause of
hypertension characterized by
paroxysms of headache, sweating
and palpitations. There are no
bruits associated with
pheochromocytoma.
(c) B. Renal artery stenosis is
characterized by hypertension
that is resistant to three or more
medications and renal artery
bruits on examination.
(u) C. Cushing syndrome is
characterized by "moon" facies, a
buffalo hump, a protuberant
abdomen, weakness and
headache. There are no renal
artery bruits associated with
Cushing syndrome.
(u) D. Coarctation of the aorta is
associated with hypertension in
the upper extremities and
normal or low blood pressure in
the lower extremities. There are
often weak femoral pulses and a
late systolic ejection murmur or
associated aortic insufficiency
murmur.

201.

Clinical Therapeutics/Cardiology
A 16 year-old male presents with complaint of syncope after basketball practice today.
Physical examination reveals a systolic murmur along the left sternal border that increases
with Valsalva maneuver. An electrocardiogram reveals left ventricular hypertrophy.
Echocardiogram shows asymmetric left ventricular hypertrophy with a hypercontractile left
ventricle. Which of the following is the initial medication of choice in this patient?
A. Metoprolol (Lopressor)
B. Losartan (Cozaar)
C. Lisinopril (Zestril)
D. Hydrochlorothiazide (Diuril)

Explanations
(c) A. Beta-blockers are the
initial drug of choice in a
symptomatic patient with
hypertrophic cardiomyopathy.
(u) B. See A for explanation.
(u) C. See A for explanation.
(u) D. See A for explanation.

202.

Clinical Intervention/Cardiology
An 80 year-old female presents with syncope and recent fatigue and lightheadedness over the
past month. She denies chest pain or dyspnea. Physical examination reveals BP 130/70 mmHg,
HR 40 bpm, regular, and RR 16. Electrocardiogram reveals two p waves before each QRS
complex. Which of the following is the treatment of choice for this patient?
A. Cardio defibrillator insertion
B. Atropine as needed
C. Permanent dual chamber pacemaker insertion
D. Ritalin therapy daily

Explanations
(u) A. Cardio defibrillators treat
ventricular tachycardia and
are not indicated in the
management of second degree
AV block.
(u) B. Atropine can be used in
the acute management of
second degree AV block Mobitz
type II, but it should not be
used as long-term therapy.
(c) C. This patient has findings
consistent with symptomatic
second degree AV block Mobitz
type II for which permanent
pacing is the treatment of
choice.
(u) D. Ritalin therapy is not
indicated in the management
of second degree heart block.

203.

History & Physical/Cardiology


Which of the following would be expected on physical examination of a newborn diagnosed
with Tetrology of Fallot?
A. Palpable right ventricular lift
B. Pulsediscrepancybetweenarmsandlegs
C. Mid-diastolic murmur with opening snap
D. Polymorphous exanthema

Explanations
(c) A. Tetralogy of Fallot is
commonly associated with a
palpable right ventricular lift.
(u) B. Coarctation of the aorta is
associated with a pulse
discrepancy between the
upper and lower extremities.
(u) C. A mid-diastolic murmur
with an opening snap is heard
in a patient with mitral stenosis,
not Tetralogy of Fallot.
(u) D. Polymorphous
exanthema is seen in patients
with Kawasaki disease.

204.

Clinical Therapeutics/Cardiology
A 67 year-old male with history of mitral valve stenosis undergoes a mechanical valve
replacement. Which of the following is the appropriate duration of anticoagulation therapy if
the patient has no other risk factors for thromboembolic events or significant bleeding risks?
A. One month
B. Three months
C. Six months
D. Lifelong

Explanations
(u) A. See D for explanation.
(u) B. See D for explanation.
(u) C. See D for explanation.
(c) D. Patients with mechanical
valves require lifelong
anticoagulation to prevent
thrombosis.

205.

Diagnosis/Cardiology
A 60 year-old female with history of radiation therapy for the treatment of
cancer presents with progressive dyspnea and fatigue. On examination the
patient has lower extremity edema, significant ascites, and an elevated
jugular venous pressure that does not fall with inspiration. Heart
examination reveals a pericardial knock. Echocardiogram shows rapid
early filling and reduced mitral inflow velocities with inspiration. Which of
the following is the most likely diagnosis in this patient?
A. Pulmonaryhypertension
B. Atrialmyxoma
C. Constrictive pericarditis
D. Tako-Tsubo cardiomyopathy

Explanations
(u) A. Pulmonary hypertension is usually associated
with chest pain, dyspnea, fatigue and syncope.
Examination would reveal a narrow splitting of S2
with a loud pulmonic component. Echocardiogram
would show increased pulmonary artery pressures,
right ventricular enlargement and possibly
paradoxical motion of the intraventricular septum.
(u) B. Atrial myxoma is characterized by fever,
weight loss, malaise, embolization, and a diastolic
sound or murmur (tumor plop) on examination.
Atrial myxoma would be seen on echocardiogram.
(c) C. Constrictive pericarditis is associated with TB,
radiation therapy, cardiac surgery, or following viral
pericarditis. There is evidence of right-sided heart
failure, a positive Kussmaul sign, and a septal
bounce and reduced mitral inflow velocities with
inspiration on echocardiogram.
(u) D. Tako-Tsubo cardiomyopathy (broken heart
syndrome) is commonly seen in postmenopausal
women who experience signs and symptoms of
acute coronary syndrome. Echocardiogram shows
left ventricular apical dyskinesia.

206.

Scientific Concepts/Cardiology
A 59 year-old male with history of hypertension and dyslipidemia presents
with complaint of substernal chest pain for two hours. The pain woke him
from sleep, does not radiate, and is associated with nausea and
diaphoresis. Electrocardiogram reveals ST segment elevation in leads II, III,
and AVF. Which of the following walls of the ventricle is most likely at risk?
A. Anterior
B. Inferior
C. Lateral
D. Posterior

Explanations
(u) A. See B for explanation.
(c) B. Inferior wall myocardial infarction is
characterized by ST segment elevation in leads II, III
and AVF.
(u) C. See B for explanation.
(u) D. See B for explanation.

207.

Diagnosis/Cardiology
A 24 year-old female presents complaining of palpitations described as
occasional "skipped" beats. The patient denies chest pain,
lightheadedness, syncope, or dyspnea. On examination you note a
midsystolic click without murmur. Which of the following is the most
likely diagnosis in this patient?
A. Mitral valve prolapse
B. Aortic stenosis
C. Atrial septal defect
D. Pulmonic stenosis

Explanations
(c) A. Patients with mitral valve prolapse will often
present with complaint of palpitations. Auscultation
would reveal a mid-systolic click with or without a
late systolic murmur.
(u) B. Aortic stenosis presents with a systolic
murmur with no click. Patients may complain of
chest pain, dyspnea or syncope.
(u) C. Atrial septal defect is not associated with a
midsystolic click.
(u) D. Pulmonic stenosis is not associated with a
midsystolic click.

208.

Health Maintenance/Cardiology
A 10 year-old female experiences fever and polyarthralgia. On examination you
note a new early diastolic murmur. Laboratory results are positive for
antistreptolysin O. The patient has no known drug allergies. Which of the
following is the recommended prophylaxis for this condition?
A. Doxycycline
B. Erythromycin
C. BenzathinepenicillinG
D. Trimethoprim/sulfamethoxazole

Explanations
(u) A. Doxycycline and Bactrim are not
indicated for the prophylaxis of recurrent
rheumatic fever.
(u) B. Erythromycin is considered second line
for prophylaxis of recurrent rheumatic fever
in a patient with a penicillin allergy.
(c) C. Recurrences of rheumatic fever are
most common in patients who have had
carditis during their initial episode and in
children. The preferred method of
prophylaxis is Benzathine penicillin G every
four weeks.
(u) D. See A for explanation.

209.

Clinical Intervention/Cardiology
A 26 year-old patient is brought to the emergency department after a head on
collision. The patient complains of chest pain, dyspnea and cough. Examination
reveals the patient to be tachypneic and tachycardic with a narrow pulse
pressure. Jugular venous distension is noted. Electrocardiogram reveals
nonspecific t wave changes and electrical alternans. Which of the following is the
most appropriate management plan for this patient?
A. Serial echocardiograms
B. Pericardiocentesis
C. Cardiac catheterization
D. Pericardiectomy

Explanations
(u) A. Serial echocardiograms would be
indicated if a patient had a small pericardial
effusion and no intervention was
immediately needed. This patient has signs
and symptoms of cardiac tamponade and
needs immediate intervention.
(c) B. Urgent pericardiocentesis is the initial
treatment of choice in a patient with cardiac
tamponade.
(u) C. There is no indication for cardiac
catheterization in the management of cardiac
tamponade.
(u) D. A partial pericardiectomy may be
needed in patients with recurrent pericardial
effusions that occur secondary to neoplastic
disease and uremia, but there is no
indication for partial pericardiectomy in the
acute management of cardiac tamponade.

210.

Health Maintenance/Cardiology
A 40 year-old G3P3003 female presents complaining of dull aching discomfort of
her lower extremities, which is worse in the evening. The patient currently works
as a waitress. Examination reveals dilated, tortuous veins beneath the skin in the
thigh and leg bilaterally. Which of the following is the best initial approach to
prevent progression of disease and complications in this patient?
A. Compression stockings
B. Warfarin (Coumadin) therapy
C. Sclerotherapy
D. Clopidogrel (Plavix)

Explanations
(c) A. Graduated compression stockings can
be used in patients with early varicosities to
prevent progression of the disease and
when used with leg elevation complications
from varicose veins can be avoided.
(u) B. There is no indication for warfarin or
clopidogrel therapy in patins with varicose
veins.
(u) C. Scierotherapy is not the best initial
choice to prevent disease progression and
complications
(u) D. See B for explanation.

211.

Diagnostic Studies/Cardiology
A 60 year-old male with history of hypertension presents for routine physical
examination. He has no current complaints. Vital signs are BP of 136/70 mmHg, HR 60
bpm, regular, RR 14, unlabored. Heart shows regular rate and rhythm with no S3, S4 or
murmur, Lungs are clear to auscultation bilaterally, and the abdomen is soft,
nontender. There is a 5cm palpable pulsatile abdominal mass noted. Which of the
following is the best initial diagnostic study in this patient?
A. Magnetic resonance imaging (MRI)
B. Arteriography
C. Ultrasound (US)
D. Plain film

Explanations
(u) A. MRI, arteriography, and
abdominal flat plate are not indicated
in the initial diagnostic evaluation of a
patient with a suspected abdominal
aortic aneurysm.
(u) B. See A for explanation.
(c) C. Abdominal ultrasound is the
diagnostic study of choice for the initial
diagnosis of an abdominal aortic
aneurysm.
(u) D. See A for explanation.

212.

Diagnosis/Cardiology
A 70 year-old female with history of hypertension, diabetes, and hypothyroidism
presents with complaint of sudden onset of left lower extremity pain. Examination
reveals a cool left lower extremity with a mottled appearance. Dorsalis pedis and
posterior tibialis pulses are absent. Which of the following is the most likely diagnosis?
A. Acute arterial occlusion
B. Thromboangiitis obliterans
C. Deep vein thrombosis
D. Peripheral neuropathy

Explanations
(c) A. Acute arterial occlusion presents
with sudden onset of extremity pain,
with absent or diminished pulses. The
extremity will be cool to the touch and
have a mottled appearance.
(u) B. Thromboangitis obliterans occurs
in younger patients and primarily
effects the distal extremities, especially
the toes. It is typically secondary to
smoking.
(u) C. Deep vein thrombosis presents
with lower extremity pain and edema.
Pulses would be intact.
(u) D. Patients with peripheral
neuropathy would have diminished
sensation. They would not have a
mottled appearance and pulses would
be intact.

213.

A 45 year-old male presents to the Emergency Department complaining of sudden


onset of tearing chest pain radiating to his back. On examination the patient is
hypertensive and his peripheral pulses are diminished. Electrocardiogram shows no
acute ST-T wave changes. Which of the following is the diagnostic study of choice in
this patient?
A. Computed tomography (CT) scan
B. Transthoracic echocardiogram
C. Magnetic resonance imaging (MRI)
D. Cardiac catheterization

Explanations
(c) A. This patient has signs and
symptoms of acute aortic dissection for
which CT scan is the diagnostic study of
choice.
(u) B CT scan is better than
transthoracic echocardiogram for the
diagnosis of acute aortic dissection.
Transesophageal echocardiogram (TEE)
is a good diagnostic modality, however
it is not always available in the acute
setting.
(u) C.MRI is good in the diagnosis of a
chronic aortic dissection, but the longer
imaging time and the difficulty in
monitoring the patient during the test
makes it not the first choice in the
setting of an acute dissection.
(u) D. Cardiac catheterization is not
indicated in the diagnosis of an acute
aortic dissection.

214.

Scientific Concepts/Cardiology
A 60 year-old male complains of progressive fatigue and dyspnea. On examination his lungs are
clear to auscultation bilaterally, heart exam reveals regular rate and rhythm without S3, S4 or
murmur, and extremities show 1+ edema bilaterally. Chest x-ray reveals cardiomegaly.
electrocardiogram shows low voltage, and echocardiogram shows an ejection fraction of 55%
with a small, thickened left ventricle that has rapid early filling with diastolic dysfunction.
Which of the following is the most likely underlying etiology of this patient's cardiomyopathy?
A. Alcoholism
B. Myocarditis
C. Amyloidosis
D. Chronic hypertension

Explanations
(u) A. Chronic alcohol use is
commonly associated with a
dilated left ventricle with left
ventricular dysfunction.
(u) B. Myocarditis is associated
with a dilated, not small, left
ventricle.
(c) C. Amyloidosis is the most
common cause of restrictive
cardiomyopathy and is
associated with a small
thickened left ventricle that
has rapid early filling with
diastolic dysfunction.
(u) D. Chronic hypertension is
associated with a
hypertrophic,
hypercontractile left ventricle.

215.

Clinical Intervention/Cardiology
A 70 year-old male with history of ischemic cardiomyopathy presents with a syncopal episode.
He denies complaints of chest pain, palpitations, or dyspnea. ECG shows no acute ST-T wave
changes. Echocardiogram reveals an ejection fraction of 25% with no valvular abnormalities.
Which of the following is the most appropriate management for this patient?
A. Dual chamber permanent pacemaker
B. Diltiazem (Cardizem)
C. Implantable cardio defibrillator
D. Midodrine (ProAmatine)

Explanations
(u) A. See C for explanation.
(u) B. See C for explanation.
(c) C. This patient has ischemic
cardiomyopathy and syncope,
which is most likely due to
ventricular tachycardia.
Instertion of a cardio
defibrillator is the
management of choice in this
patient.
(u) D. See C for explanation.

You might also like